SlideShare a Scribd company logo
1 of 75
Download to read offline
OMPREHENSION
Q.1-10. Read the following passage carefully and answer the questions given below it.
You’ve got a sarkari naukri, a government job, whether you know it or not. Or whether you like it or
not. Sarkari naukri doesn’t mean that you have to be a government babu, or an MLA, or MP, or a
member of the judiciary or of the defence forces, or an employee of a PSU. You can be a teacher, or
an accountant, or a marketing executive, or an event manager, or a homemaker, or a student, but you
are still a full-time government employee. By sarkari naukri I’m not talking about the much-touted and
equally criticised NREGS. The sarkari service that compulsorily we all do, whether we know it or not,
is - unlike NREGS - purely honorary, in the sense that we don’t get paid for it. On the contrary, all of
us unwitting government servants, in one way or the other, pay the government for the privilege of our
having been given an unpaid government job to do. If you belong to the 2% of the population of this
country which pays income tax you are an unpaid sarkari servant. For if you have a taxable income
you are not only obliged to pay a tax on it but you are also obliged to file returns to prove that you
have indeed paid the tax due from you. In other words, you not only pay your tax but you also become
an unpaid tax assessor-cum-collector for the government because the onus of proof is on you to show
that not only have you paid your tax but that you have done the government’s work of having updated
the government’s records which show you’ve paid the tax. Unless you are an accountant yourself, you
have to pay an accountant to file your tax returns for you. (Have you ever tried to figure out a tax
return form? Instead of SAT and GMAT and all those other tests to gauge students’ mental abilities
they should ask candidates to fill in an average sarkari tax form. Any non-specialist who can do this
has a genius-level IQ and is a potential member of Mensa.) So you’ve not only paid your tax, and also
done the government’s unpaid work of filing your returns, but on top of that you’ve shelled out an
accountant’s fee for having done the government’s job for it. You’ve paid for the privilege of having
sarkari naukri. OK, so you don’t have a taxable income. No matter. You’re still eligible for sarkari
naukri. Each time you drink a cup of tea or coffee, travel by bus or car, go for a medical check-up, buy
anything from chalk to cheese, from an apple to a zither and anything in between you’re raising
revenues for the government through a spiderweb of taxes including sales taxes, excise duties, octroi,
et taxcetera, et taxcetera. Which means that you are an unofficial employee of the sarkar (why else
would you be raising funds for it?), which means you are a sarkari naukar-chaakar. The sarkar taxes
almost everything of daily use, for all of us, from the richest to the poorest. What was that you said?
The only thing left untaxed is love and fresh air? Uh, oh. You might have just given the FM a couple of
taxation ideas for his next budget. Our sarkar’s motto is: Ask not what your sarkar can do for you; ask
what you can do for your sarkar. As more and more of us become the unpaid slaves of the sarkar, the
sarkar itself does even less and less. Which means that we, the sarkari karamcharis, have to do more
and more. And our sarkari naukri is for life. There’s no such thing as retirement. And that’s a sarkari
guarantee, like it or lump it. Lump is more like it.
Q.1. What is Sarkar’s motto ?
(1) To ensure a good living for every citizen
(2) To ask what a citizen can do for it, not what it can do for the citizens
(3) Levy taxes on the commodities
(4) Raise the standard of living
(5) None of these
Q.2. How does the author say that one can be an “unpaid Sarkari Servant” ?
(1) By paying income tax regularly
(2) If we are among 2% of the Indian population.
(3) By shelving the accountant’s fee for filing returns.
(4) All 1,2 and 3
(5) Only 1 and 2
Q.3. How does the author say that even if you do not have a taxable income you are a “Sarkari
Servant” ?
(1) By traveling by bus or car
(2) By raising revenues
(3) By paying indirect taxes on the commodities we use or buy.
(4) All 1,2 and 3
(5) None of these
Q.4. How according to the author is sarkari naukri for life ?
(1) Because there is a sarkari guarantee for us.
(2) As there is no retirement in this job.
(3) As the job offers us life time service
(4) Because the government will continue taking taxes from us
(5) None of these
Q.5. How is paying tax also becoming an unpaid tax assessor-cum-collector for the government ?
(1) By being a part of the NREGS
(2) As by paying tax we are indirectly updating the government’s records.
(3) By being an accountant yourself
(4) By filing your returns yourself
(5) All of the above
Q.6. Which of the following are the taxes mentioned in the passage ?
(1) Sales taxes, excise, duties, octroi, et taxcetra , etaxcetra
(2) Sales tax, excise duties octroi, et taxcetra
(3) Sales tax, excise duties, octroi
(4) Sales tax,exicise duties, octroi, extaxcetra
(5) Sales tax, excise duties, et taxcetra
Q.7. What does the author show by this passage ?
(1) Importance of government services
(2) Taxes and their role in the country
(3) Heavy burden of taxes on all the citizens
(4) All 1,2 and 3
(5) Only 2 and 3
Q.8. What does the following statement mean – “which means that, we, the Sarkari Karamcharis,
have to do more and more” ?
(1) The government servants have to work harder
(2) The citizens have to pay more taxes to the government every year.
(3) We the citizens of India have to pay taxes regularly
(4) All 1,2 and 3
(5) None of these
Q.9. Who are the “unpaid slaves” ?
(1) People in government services
(2) Citizens of the country
(3) Each and every citizen of the country
(4) Persons working under the NREGS scheme
(5) People who are not necessarily in the government service but pay taxes.
Q.10. Which of the following is TRUE is context of the passage ?
(1) 2% of the population of our country is under the tax free income
(2) Indian government levies taxes on all the commodities
(3) People who are not officially government servants are free from tax.
(4) All the citizens are full time government servants indirectly
(5) None of these
ANSWERS:
Q.1.(2)
Q.2.(1)
Q.3.(3)
Q.4.(4)
Q.5.(2)
Q.6.(3)
Q.7.(2)
Q.8.(5)
Q.9.(5)
Q.10.(5)
---------------------------------------------------------------------------------------------------------------------------------------
--------------------------------------------
FILL IN THE BLANKS
Q.1-5. In each of the following sentences there are two blank spaces. Below each five pairs of words
have been denoted by numbers (1), (2), (3), (4) and (5). Find out which pair of words can be filled up
in the blanks in the sentences in the same sequence to make the sentence meaningfully complete.
Q.1. Despite their strong resentment, the tribals ———— the new laws as long as the government
officials did not ———— them too strenuously.
(1) welcomed, observe (2) tolerated, enforce (3) amended, follow (4) laid, punish (5)
absorb, effect
Q.2. He gave me an additional ——— of Rs. 200 a month by ——— of the commissioner.
(1) allowance, permission (2) pay, allowance (3) salary, admittance (4) payment, agreement
(5) benefit, laws
Q.3. He was too ———— to ————— that he had been wrong.
(1) docile, accept (2) polite, agree (3) nervous, rejoice (4) obstinate, admit (5)
determined, promise
Q.4. I am not —— to sell you my house unless you offer a more ——— price.
(1) conform, true (2) prepared, realistic (3) ready, correct (4) great, achieved (5)
accepted, standard
Q.5. Since we ——— read every book, we ——— only the famous ones.
(1) have, sold (2) should, buy (3) must, ignore (4) can, purchase (5) cannot, select
ANSWERS
Q.1.(2) Tolerated here means beared the laws and enforce means brought into action or force.
For other options:-
- Welcomed means accepted openly or happily and observe means to study something closely.
- amended means changed, it can not be used here as the tribals can not change the laws.
- laid means mode or brought into form. The tribals will not lay the laws of the government.
Q.2.(1) allowance means a sum of money granted for a particular purpose and the word permission
is compatible with it. The other options pay, salary and payment can not be used here with the word
additional.
Q.3.(4) obstinate means firm or stubborn, generally used in a negative sense and admit means
accept.
For other options:-
- docile means easily managed or handled it is inappropriate here.
- determined means dedicated towards something but the word promise is not suitable in the latter
part of the sentence.
- polite and nervous do not present a negative sense here so they cannot be used.
Q.4.(2) Prepared means ready mentally and realistic means having a sensible and practical aspect.
Q.5.(5) Cannot means not able to and select means choose, as we cannot read every book so we
select only the famous ones.
---------------------------------------------------------------------------------------------------------------------------------------
--------------------------------------------
SENTENCE COMPLETION
Q.1-5. In each of the following sentences there is a blank space given. Below each statements have
been denoted by numbers (1), (2), (3), (4) and (5). Find out which statement can be filled up in the
blank in the same sequence to make the sentence meaningfully complete.
Q.1. BRICS is a group of emerging economies of the world. The word BRIC was originally coined by
JIM O’meiu of Goldman Sachs in 2001. Goldman Sachs is an international financial company which
conducts research in the future growth of global economy. ___________
(1) Global economy generally refers to the economy, which is based on economies of all of the worlds
countries.
(2) Goldman Sachs has identified Brazil, Russia, China and India as the four rising economies.
(3) It appears that South Africa has been included in BRICK due to strategic regions.
(4) Research and development is and integral part of every organization.
(5) Optimism is difficult when considering an unpredictable future.
Q.2. It has become the latest fashion not only in political circles but also in print and media circles to
blame Armed Forces Special Powers Act for all the unrest turbulence ———— very. Conveniently
they all overlook the undeniable fact that Pakistan after suffering a crushing defeat from India in 1971
has disclosed an open proxy war against India.
(1) and large scale violence not only in just Kashmir valley but also in many other parts like Manipur.
(2) Parliament after serious deliberations want ahead to enact the AFSPA on September 11,1958 to
safeguard the armed Forces.
(3) AS terrorism raise its ugly head in Jammu and Kashmir in late eighties, Parliament is self and not
Army felt compelled to enact the act.
(4)The AFSPA was being unnecessarily demonized although it had nothing to do with the present
unrest in Kashmir.
(5) The Armed Forces are not only deployed to work in hostile environment but also in a difficult
terrain.
Q.3. You must create a clear vision of your target and never lose sight of it.
-————. Your incentive loses appeal, efforts slacken and you make no progress.
(1) Psychological laws usually are not so dramatic.
(2) The practice of keeping your eyes open is complicated.
(3) Once you allow your gaze to wander, good intention fade.
(4) They gather details of anybody’s private life for the safe purpose of passing on scandalous tidbits
to others.
(5) In everyday conversation, we speak of looking forward to some pleasure.
Q.4. While preparing a presentation, the beginning should be carefully designed to get attention. A
shock, a story, humor, question or facts and figures can achieve this purpose for you. Howewer, you
should be so confident and well rehearsed that they look natural ——
(1) The key points in the middle section of your presentation should be logical.
(2) Ensure that your are audible to all the audience.
(3) Keep your comments brief and avoid being caught extempore at only point or question.
(4) Make sure that your visual aid is clearly visible and legible to all the audience.
(5) By mixing them at intervals in your presentation you will be able to hold their interest and
communicate your message successfully.
Q.5. ——————. It deals with those aspects even if it is minor or major but essential for building the
constitution. If a person has studied it deeply he will be able to understand instance of constitution.
(1) What is of supreme importance is that parliamentarians must be totally honest.
(2) Politics is very crucial part of nation as it involves each person of society for framing any law,
policy or framework.
(3) India being a democratic nation has the viability of accepting the right and efficient person as their
representative and it must be fulfilled for welfare of people.
(4) A member of parliament should be at least post graduate and that too in Human Resources or
Marketing.
(5) A public representative ought to have sensible mind with high degree of zone of tolerance and
attitude of positive progressive thinker.
ANSWERS
Q.1.(2) The passage is talking about BRICK and how the concept come up along with stating what
Goldman sachs is.
Q.2.(1) The writer is talking about AFSPA and how it is perceived by the society at large.
Q.3.(3) The author is trying to tell us how we should have a clear picture of our target and never loose
over determination.
Q.4.(5) Here it is being told how one should prepare and deliver a presentation and the various ways
to make it effective.
Q.5.(2) The writer speaks on Politics, its pros and cons and what a person can gain by learning it.
---------------------------------------------------------------------------------------------------------------------------------------
--------------------------------------------
SENTENCE IMPROVEMENT
Q.1-5. Which of the Phrases (1), (2) (3) and (4) given below each sentence should replace the
phrases printed in bold in the sentence to make it grammatically correct ? If the sentence is correct as
it is given and no correction is required, mark (5) as the answer.
Q.1. Of the various sectors of the Indian economy, the agriculture sector is the one that have large
witnessing the era of modernization.
(1) have large witnessed (2) was large witnessing (3) were largely witnessed (4) has
largely witnessed (5) No correction required
Q.2. The primary objective of the association is being overthrow the British Raj in India and establish
a national republic passed on freedom and equality.
(1) was being (2) to be (3) was to (4) were to be (5) No correction required
Q.3. The past two decades see several attemptings to set up economic alliances, but they all failed.
(1) see several attempt (2) saw several attempts (3) view several attempted (4) see
several attemps (5) No correction required
Q.4. The report suggested that it was only those at the bottom of the economic ladder who suffered
through the recent recession years.
(1) who were suffered (2) who was suffered (3) who is suffered (4) who for suffered
(5) No correction required
Q.5. During the crises, the prompt actions of policymakers across the globe was for being critical in
averting a much deeper disaster.
(1) was critical (2) were critical (3) to critical (4) being (5) No correction required
ANSWERS:
Q.1.(4)
Q.2.(3)
Q.3.(2)
Q.4.(5)
Q.5.(2)
---------------------------------------------------------------------------------------------------------------------------------------
--------------------------------------------
ERROR DETECTION
Q.1-5. Read each sentence given below and find out whether there is an error in it. The error if any will be one
of the parts of the sentence which are marked as 1,2,3 and 4. If there is no error, the answer will be
(5) i.e. No error. (Ignore the errors of punctuation, if any)
Q.1. After finishing(1)/ his speech(2)/ the audience was invited(3)/to ask questions(4)/. No error(5)
Q.2. The little child(1)/was sitting closely(2)/beside(3)/his teacher(4)/. No error(5)
Q.3. Built on 230 acres(1)/the palace of Udaipur(2)/is one of the (3)/showplaces of Rajasthan(4)/. No
error(5)
Q.4. Tonight(1)/ I am going to check(2)/that Sanjeev(3)/will do his homework correctly(4)/. No error(5)
Q.5. Due to the house(1)/ sparkled with flames(2)/the fireman used the water(3)/to extinguish the
fire(4)/. No error(5)
ANSWERS
Q.1.(5) No error.
Q.2.(2) Use ‘close’ in place of closely here
Q.3.(5) No error
Q.4.(4) This sentence talks about the person ensuring that Sanjeev does his work correctly
For personal plans or predictions we use “going to” with the simple present tense.
Q.5.(1) Use “as” in place of due to, “Due to” is followed by a noun.
--------------------------------------------------------------------------------------------------------------------------------------
---------------------------------------------
SENTENCE EVALUATION
Q.1-4. In each of these questions there are three sentences given as(I),(II),(III),find out which two or
three sentences convey the same meaning.If all three sentences are different, mark (5) as the answer
i.e ‘all different’.
Q.1. I. The Manager would like you to help him in locating the fault.
II. If you help him in locating the fault , the Manager would like you.
III. The Manager desires that you should provide him the necessary assistance to locate the fault.
(1) I and II (2) II and III (3) I and III (4) All are same (5) All are different
Q.2. I. Although strike of transporters continues, I shall come.
II. I shall come if the strike of transporters continues.
III. Even though I come, the strike of transporter is going to continue.
(1) I and II (2) II and III (3) I and III (4) All are same (5) All are different
Q.3. I. Unlike the tribals who are hard working the urban communities cannot withstand physical
strain.
II. The tribals can withstand physical strain where as urban communities cannot.
III. Because the tribals are hardworking they can tolerate physical strain.
(1) I and II (2) II and III (3) I and III (4) All are same (5) All are different
Q4. I. If you need a visa ,you must submit an application along with your passport and a copy of
income tax returns.
II. Unless you submit an application along with your passport and copy of income tax returns you will
not get visa.
III. If you submit your application along with your passport and a copy of income tax returns you do
not need visa.
(1) I and II (2) II and III (3) I and III (4) All are same (5) All are different
ANSWERS
Q.1.(3) Both (1) and(3) convey the same meaning
Q.2.(5) All the three statements convey different messages.
Q.3.(5) All the three statement have different meaning so (5)is the answer.
Q.4.(1) Both (1)and (2) sentences give the same meaning that for getting visa, passport and copy of
income tax returns are mandatory.
---------------------------------------------------------------------------------------------------------------------------------------
--------------------------------------------
ERROR DETECTION
Q.1-5. Read each sentence to find out if there is any error in it. The error, if any, will be in one part of
the sentence. The number of that part is the answer. If there is no error, the answer is (5).
Q.1. If you will buy one box(1)/ at the regular price(2)/ you will receive another(3)/ at no extra cost (4)/
No Error (5)
Q.2. The train(1)/ has arrived (2)/ when he reached(3)/ the station (4) /No Error (5)
Q.3. Daniel was(1)/one of the greatest(2)/ judges that(3)/has ever lived(4)/ No Error (5)
Q.4. The climate of(1)/ Goa is (2)/more pleasant(3)/ than Inland resort(4) / No Error (5)
Q.5. She presented (1)/ him(2)/two dozens (3)/pens(4)/ No Error (5)
ANSWERS
Q.1.(1) "Will" should replace omitted because if is not followed by "will".
Q.2.(2) "Had " should replace"has" because the sentence is in past tense.
Q.3.(4) "Have" should replace "has" because one of than Noun (plural) who/which/ that+verb(plural).
Q.4.(4) "Than" should replace followed by "than of ".
Q.5.(3)
---------------------------------------------------------------------------------------------------------------------------------------
--------------------------------------------
FILL IN THE BLANKS
Q.1-5. Fill in the blanks with appropriate words in both the blanks
Q.1. The petitioner had ____ an immediate stay from the court on allotment of the flats till ____ of
investigation into the alleged irregularities.
(1) asked, process (2) sought, completion (3) propagated, finish (4) demanded, course
(5) granted, end
Q.2. Modern ‘music, doesn’t lend ______ to longevity; Sometimes it is hand to remember what the
biggest hit from a _____ of years ago was.
(1) itself,couple (2) must, few (3) money, some (4) matter, spatter (5) substance,
vacant
Q.3. Opportunities_____ when they are seized and _____when they are not .
(1) gather, evade (2) needed, reflect (3) create, disperse (4) shout, calm (5) multiply,
die
Q.4. At the sound of bell we are ____to pause, relax our body and become gently _____of our in-
breath and out-breath.
(1) asked, thoughtless (2) requested, wishful (3) invited, aware (4) forget, meaningful
(5) braced,calm
Q.5. ______to listen to the sounds of nature, we find that we can afterwards carry on with whatever
we were doing in a more _____ and loving way.
(1) feeling, helpful (2) waiting, rough (3) dreaming, dramatic (4) pausing, attentive (5)
hurting, creative
ANSWERS
Q.1.(2)
Q.2.(1)
Q.3.(5)
Q.4.(3)
Q.5.(1)
---------------------------------------------------------------------------------------------------------------------------------------
--------------------------------------------
REARRANGEMENT
Q.1-5.Rearrange the following sentences (A),(B),(C), (D), (E) and (F) in the proper sequence to form a
meaningful paragraph, then answer the questions given below them.
(A) The students were introduced to make a working model of cars using solar energy
(B) The students were elated after successfully completing the working model of the car
(C) But in a matter of one hour, their nimble hands transformed
(D) At a time when the significance of research activity shifted to the field of clean energy
(E) These parts into small and beautiful cars that were powered by solar energy.
(F) They were given wires, plastic parts, tiny wheels and solar panels
Q.1. Which of the following would be the SECOND statement after rearrangement?
(1) C (2) A (3) D (4) F (5) E
Q.2. Which of the following would be the SIXTH statement after rearrangement?
(1) A (2) F (3) D (4) B (5) C
Q.3. Which of the following would be the THIRD statement after rearrangement?
(1) C (2) A (3) D (4) F (5) E
Q.4. Which of the following would be the FIRST statement after rearrangement?
(1) C (2) F (3) E (4) B (5) D
Q.5. Which of the following would be the FIFTH statement after rearrangement?
(1) E (2) A (3) D (4) B (5) C
ANSWER
Q.1.(2)
Q.2.(4)
Q.3.(4)
Q.4.(5)
Q.5.(1) The arrangement is DAFCEB The paragraph is about an assignment which was given to
students for making cars using solar energy which brought them contentment and happiness
---------------------------------------------------------------------------------------------------------------------------------------
--------------------------------------------
FILL IN THE BLANKS
Q.1-5.In each of the following sentences there are two blank spaces. Below each five pairs of words
have been denoted by numbers (1), (2), (3), (4) and (5). Find out which pair of words can be filled up
in the blanks in the sentences in the same sequence to make the sentence meaningfully complete.
Q.1. Infrastructure services -----------to poverty reduction and improvements in living -------in several
ways.
(1) confer, segment (2) contribute, standards (3) subsidize, style (4) extend , quality (5)
regulate, organisms
Q.2. The organic mangoes are------------- without using any chemical manure, pesticide, but are --------
---naturally.
(1) flourished, seasoned (2) evolved, flowered (3) grown-up, developed (4) matured,
bloomed (5) grown, ripened
Q.3. Hafiz ----------- faces and sells flags branded with the IPL teams to -----------a meal for his family
(1) paints, earn (2) makes, manages (3) coloured, obtain (4) drew, secure (5) read,
receive
Q.4. Groups of auto lifters were -----------and found to have changed the colour of ---------bikes.
(1) spotted, repair (2) exposed, borrowed (3) seized, bought (4) caught, stolen (5)
grabbed, maintained
Q.5. People --------difficulties in reaching their destinations on ---------- due to traffic jam
(1) suffered, travel (2) challenged, arrival (3) faced, time (4) experienced, demand (5)
sustained , crow
ANSWERS
Q.1.(2) Contribute, standards
For other options:
Confer (v)- give
Subsidize (v)-contribute
Regulate (v)- control
Q.2.(5) Grown, ripened
For other options:
Seasoned (v)- made tasty
Evolved (v)-emerged
Q.3.(1) Paints, earn
Q.4.(4) Caught, stolen
Q.5.(3) Faced, time
---------------------------------------------------------------------------------------------------------------------------------------
--------------------------------------------
ERROR DETECTION
Q.1-5. Read each sentence to find out if there is any error in it. The error, if any, will be in one part of
the sentence. The number of that part is the answer. If there is no error, the answer is (5).
Q.1. Police are keeping an eye on the top notch bookies (1)/ as they would get evenly more active (2)/
with two matches of IPL (3)/ taking place in the city (4)/ No error (5)
Q.2. The saplings, kept in nurseries (1)/ are ageing and the roots (2)/ are begin to grow out of plastic
containers (3)/ and getting into the ground (4)/ No error (5)
Q.3. Fire personnel tried its best (1)/ to prevent spread of the fire (2)/ in the congested locality
(3)/adjacent to the affected godown (4)/ No error (5)
Q.4. It is in the interest of the customers to provide (1)/ their contact address and telephone numbers
(2)/ so that the banks can pass on any information (3)/ about their services and products (4)/ No
error(5)
Q.5. The bamboo umbrellas, which used to shelter pundits and sages (1)/ on the banks of Ganga (2)/
are being replaced artificial umbrellas (3)/ made up of plastic (4)/ No error (5)
ANSWERS
Q.1.(2) ‘Even’ should replace ‘evenly’- an adverb only would qualify another adverb ‘more’
Q.2.(3) ‘Beginning’ should replace ‘begin’ – present continuous tense is to be used
Q.3.(1) ‘Their’ should replace ‘its’ as possessive pronouns according to ‘fire personnel’
Q.4.(5)
Q.5.(3) ‘Replaced with’ should be used- ‘replaced’ is always followed by ‘with’
---------------------------------------------------------------------------------------------------------------------------------------
--------------------------------------------
MISAPPROPRIATION
Q.1-5.In each question below a sentence with four words printed in bold type is given. These are
numbered (1),(2),(3) and (4) . One of these four words printed in bold may be either wrongly spelt or
inappropriate in context of the sentence. Find out the word which is wrongly spelt or inappropriate if
any. The number of that word is your answer. If all the words printed in bold are correctly spelt & also
appropriate in the context of the sentence, mark (5) "All correct" as your answer.
Q.1. Without any prior notice to the traders, a portion of the road has been barricaded to
(1) (2) (3)
undertaken repair work All correct
(4) (5)
Q.2. The elephant had fallen sick the day after being tranquilized by foresters and veterinary
(1) (2) (3)
doctors were called in to treat it All correct
(4) (5)
Q.3. Young girls as well the grown-up ones are now looking to learn various techniques to keep
(1) (2) (3)
themselves protected All correct
(4) (5)
Q.4. Some of the people running small sideroad businesses, like taking children for horse rides,
(1) (2) (3)
are misbehaving with kids All correct
(4) (5)
Q.5. It requires highest measures of creativity to manufacture a play which engages the attention
(1) (2) (3) (4)
and thoughts of the viewers All correct
(5)
ANSWERS
Q.1.(4) ‘Undertake’ should be used – after ‘to’ first form of verb is to be used
Q.2.(5)
Q.3.(1) ‘As well as’ should be used which means ‘also’
Q.4.(2) ‘Roadside’ is the correct word
Q.5.(2) ‘Produce’ should be used – which would mean ‘present a play’
---------------------------------------------------------------------------------------------------------------------------------------
--------------------------------------------
READING COMPREHENSION
Q.1-10. Read the following passage carefully and answer the questions given below it.
The Aadhaar scheme of the Unique Identification Authority of India (UIDAI) is to provide India’s billion-
plus people with a unique identification number. Enrolment is not mandatory, though it was mentioned
that it would be difficult for people to access public services if not done. The scheme requires
individuals to provide their photograph, fingerprints and iris scan along with documentary personal
information for data capture by outsourced operators. It is meant to bypass the corrupt bureaucratic
system and deliver government subsidies and grants to the poor, and bring them into the banking
system. Sceptics argue that it is an effort to capture the funds of hundreds of millions of micro- and
nano-investors who are today outside the banking system, to bring them into the credit economy.
The scheme was introduced as a pilot project in Karnataka’s Mysore district. The poor and those who
survive on daily wages were not enthusiastic about enrolment, because it meant losing four or five
days wages, to stand in queues, to fill up forms, to produce documents, to provide biometrics, etc.,
and, later, to open bank accounts. The UIDAI overcame the initial reluctance by wide advertisement
of the benefits of enrolment. When this too did not achieve the target set, the local administration
informed the public that PDS ration and LPG supply would not be available without the Aadhaar
number. This resulted in serpentine queues right through the day at enrolment centres, at the end of
which the UIDAI could claim that 95 per cent of Mysore district’s population had enrolled itself into the
scheme.
Media reports indicate that commencing January 1, 2013, MGNREGA, the Rajiv Gandhi Awas Yojana
(RGAY), the Ashraya housing scheme, Bhagyalakshmi and the social security and pension scheme
will be linked with Aadhaar in Mysore district. This linking, with rights like salary and pension, and
important entitled benefits and services, has raised some hackles because enrolment is not
mandatory.
It has led to questions on whether salary and pension rights, and benefits like PDS ration and LPG
supply can be denied just because an individual does not possess a unique Aadhaar number. Today,
teachers in Maharashtra and government employees in Jharkhand cannot draw their salaries. Apart
from pro-poor projects like MGNREGA and RGAY, even jobs, housing, provident funds and
registering a marriage now require enrolment. From being not mandatory, the “poor-inclusive”
.Aadhaar scheme appears to have quietly metamorphosed into becoming exclusionary and non-
optional.
The UIDAI’s own Biometrics Standards Committee stated that retaining biometric efficiency for a
database of more than one billion people “has not been adequately analysed” and the problem of
fingerprint quality in India “has not been studied in depth.” Thus the technological basis of the project
remains doubtful.
Q.1. Which of the following are required for enrolment for ‘Aadhar scheme’?
A. Personal information
B. Iris scan
C. Photograph
D. Fingerprints
(1) Only A and C (2) Only B and D (3) Only B and C (4) All except A (5) All of the above
Q.2. Why, according to the author, are the poor not interested in ‘Aadhar scheme’?
(1) They are not willing to provide their biometric measures
(2) They do not possess the required documents which are needed to be produced at the time of
enrolment
(3) They do not perceive it as a benefit to them
(4) It involves a number of procedures which are to be completed and they will have to sacrifice their
daily work schedule
(5) The poor are interested in enrolling into ‘Aadhar scheme’
Q.3. Which of the following questions has been raised by the author?
(1) Why did ‘Aadhar scheme’ not prove to be useful?
(2) Can a person be denied benefits like PDS ration and LPG supply because he does not have
Aadhar card?
(3) Why is Aadhar enrolment not mandatory?
(4) Why can’t government employees in Jharkhand draw their salary?
(5) No question has been raised by the author
Q.4. What is meant by ‘Biometric efficiency’ as mentioned in the passage?
(1) Correct biometric measurement
(2) Biometric analysis
(3) Personal efficiency
(4) Biometric system
(5) Biological specifications
Q.5. Which scheme is NOT linked with Aadhar scheme in Mysore district?
(1) Rajiv Gandhi Swarna Jayanti swarojgar yojna
(2) Ashraya housing scheme
(3) Social security and pension scheme
(4) MGNREGA
(5) All of the above
Q.6. ———————— does not require Aadhar enrolment.
(1) Housing
(2) Admission in college
(3) Registering a marriage
(4) Provident funds
(5) Jobs
Q.7. What was declared initially about ‘Aadhar scheme’?
(1) One can get enrolled into ‘Aadhar’ online and Aadhar card would be uploaded on the website
(2) One can draw pension or salary only when one has Aadhar card
(3) Aadhar card would serve to prove one’s identity and is nontransferable
(4) One would not get access to public services if he does not get enrolled into ‘Aadhar scheme’
(5) Not mentioned in the passage
Q.8. Which of the following is FALSE according to the passage?
(1) Fingerprint quality in India has not been studied properly
(2) Aadhar scheme was introduced to provide with a universal identification number
(3) Technological basis of the project cannot be established
(4) The scheme was introduced as a pilot project in Karnataka’s Mysore district
(5) All of the above
Q.9. What led UIDAI to claim that 95 per cent of Mysore district’s population had enrolled itself into
the scheme?
(1) When teachers and government employees could not draw their salaries
(2) When benefits of Aadhar were publicized widely
(3) When it was declared that PDS ration and LPG supply would be available to only Aadhar card
holders
(4) When the queues at enrolment centres became longer
(5) All of the above
ANSWERS:
Q.1.(5)
Q.2.(4)
Q.3.(2)
Q.4.(1)
Q.5.(1)
Q.6.(2)
Q.7.(4)
Q.8.(2)
Q.9.(3)
---------------------------------------------------------------------------------------------------------------------------------------
--------------------------------------------
CLOZE TEST
Q.1-10. In the following passage there are blanks each of which has been numbered. These numbers
are printed below the passage and against each five words have been suggested, one of which fits
the blanks appropriately. Find out the appropriate word in each case.
A student looking for personal attention ends up in a tutoring class that is no less crowded. With no
regulation, the (1) could go up to a hundred. Many coaching institutions hesitate to include “average”
students. Like a shadow, private tuitions go on in tandem from KG to overseas degrees. Private
coaching institutions (2) the prevailing demand for education. Their success comes from the
government’s “inability to provide education to everyone who desires it, provide education of quality
perceived to be (3).” These centres promise students the limited seats in credible higher institutions
and social support for them is pledged by parents (4) to meet the high cost Many parents are unable
to help children with home work at specialised levels. Added to these, are the lengthy curriculum,
scarcity of efficient teachers, communication gap between teachers and students, disturbed academic
schedules, co-curricular activities that takes the students away from classrooms, tuitions that are seen
as a status symbol, (5) of some subjects over others, faulty school administration in allotting classes
and faulty government policies. Private coaching seems the only (6) Teaching is bad in general and,
coaching ensures (7) the exams. Private tutors keep students busy during off hours and, (8) unruly
ones. If such centres are bad, why haven’t the authorities taken (9) steps? Compared to a lot of
schools, aren’t coaching institutes better in terms of teaching, dedication of instructors, student-
teacher relationship and norms of conducting mock tests? In the current academic environment,
private tuitions are a practical (10). Though there seems to be varied reasons for students to seek
private tutoring, the predominant one is the need (or craze) to score better.
Q.1. (1) strength (2) reach (3) crowd (4) intelligence (5) expertise
Q.2. (1) maneuver (2) impose (3) ditch (4) exploit (5) debilitate
Q.3. (1) economical (2) pleasing (3) questionable (4) detrimental (5) worthwhile
Q.4. (1) incline (2) willing (3) leave (4) opt (5) hesitant
Q.5. (1) partially (2) improvement (3) preference (4) alternative (5) blessing
Q.6. (1) solution (2) way (3) clarification (4) feedback (5) complication
Q.7. (1) preparing (2) passing (3) through (4) developing (5) excelling
Q.8. (1) cherish (2) submissive (3) temper (4) terminate (5) tame
Q.9. (1) valid (2) agile (3) effective (4) restless (5) haste
Q.10. (1) essence (2) excuses (3) constraint (4) necessity (5) obsessed
ANSWERS
Q.1.(1) Strength
For other options:
Expertise (n)- skill
Q.2.(4) Exploit
For other options:
Maneuver (v)-plan
Impose (v)-dictate
Debilitate (v)-exhaust
Ditch (v)-elude
Q.3.(5) Worthwhile
For other options:
Economical (adj)-frugal
Detrimental (adj)-damaging
Q.4.(2) Willing
For other options:
Opt (v)- choose
Hesitant (adj)-reluctant
Q.5.(3) Preference
For other options:
Partially (adv)- not completely
Alternative (n)- choice
Q.6.(1) Solution
For other options:
Clarification (n)-description
Feedback (n)-response
Q.7.(2) Passing
Q.8.(5) Tame
For other options:
Submissive (adj)-compliant
Cherish (v)- enjoy
Temper (n)-attitude
Q.9.(3) Effective
For other options:
Agile (adj)- quick
Restless (adj)- untiring
Valid (adj)- logical
Q.10.(4) Necessity
For other options:
Essence (n)-significance
Obsessed (adj)-captivated
---------------------------------------------------------------------------------------------------------------------------------------
--------------------------------------------
SENTENCE COMPLETION
Q.1-4.In each of the following questions a short passage is given with one of the lines in the passage
missing and represented by a blank. Select the best out of the five answer choices given to make the
passage complete and coherent.
Q.1. Sipping endlessly from that hot mug of tea or coffee might soon be a thing of the past with
climate change. Tea is dependent on the distribution of rainfall and its pattern. The impact of global
warming is immense and there is a steady decline in productivity,—————————. Tea production
is mostly helped by land cover, but climatic factors have been playing spoilsport for a while.
Secondary variables such as pests and diseases and management policies too play deterrents.
(1) It has not affected coffee production so far as the crop is largely grown under the shade of trees
(2) On an average, 108 million kg tea is contributed by South Indian tea gardens, as per records
(3) Elsewhere in the world, climate change is likely to drastically affect the production of coffee
(4) However, all tea-growing regions across the world are experiencing adverse climatic and weather
conditions
(5) Where agricultural practices depend on rain-fed conditions
Q.2. Police have started filing FIRs against overloaded heavy vehicles —————————————
——. Police have launched this initiative to control accidents caused by heavy vehicles that are found
responsible in most road accident cases. Overloaded vehicles lead to destruction of road surfaces
which in turn cause traffic jams and accidents.
(1) Those heavy vehicles were rashly driven and overloaded beyond their capacity
(2) Driving through expressway during night is fraught with danger due to the heavy vehicles
(3) Under the Prevention of Damage to Public Property (PDPP) Act and several sections of the India
Penal code (IPC).
(4) The first week of April, cops have been filing FIRs against three-four dumpers on a daily basis.
(5)They indulge in speeding and frequent lane changes to reach their destinations.
Q.3. Bharathi Lions Talking Library is a treasure trove not just for visually-challenged school and
college students but also for researchers and professionals. More than 50 visually-challenged people
visit the ‘talking library’ every week to get the recorded cassettes or get their notes recorded. ————
——————————————————-The library, which opened five years ago, has around 1,300
cassettes on school and college subjects.
(1) Books recorded at the talking library so that he could listen to them later
(2) Without learning Braille, the students will not be able to understand spellings and the language
which will be crucial in their career later
(3) We either recite the contents or depute volunteers for the task
(4) There is a librarian who records it for them or gets someone else to record it for them.
(5) With advent of many technological tools to help the visually-challenged students, experts say,
learning of Braille is slowly becoming a passé
Q.4. —————————————————————————————, child counsellors suggest
them to mend the same in order to keep a check on their wards behavioral development and outlook
towards life in general. With the younger generation spending a good amount of their time on virtual
space, parents have much to catch up on their kid’s life. Until and unless that gap between the kid
and the parents isn’t bridged, wherein parents are supposed to talk about anything and everything
with their children, kids will never ever confide themselves
(1) While teenage girls are easy to communicate with, boys are usually a hard nut to crack.
(2) Citing communication gap as a major reason of divide between children and their parents
(3) They should not impose opposing views directly on to the kids
(4) The idea is to make the kid talk first and then take them on the path of persuasion
(5) We need to be their friends and then engage in an open communication with them
ANSWERS:
Q.1.(5) The paragraph is about the effect of global warming on tea production
Q.2.(3) The paragraph is about the new initiative of police to check overloaded heavy vehicles
Q.3.(4) The paragraph describes the services provided by Bharathi Lions Talking Library for visually-
challenged people
Q.4.(2) The paragraph described child counselors suggestion to bridge the gap between children and
their parents
---------------------------------------------------------------------------------------------------------------------------------------
--------------------------------------------
VOCABULARY
Q.1-4.Below is given a single word with options to its meaning. You have to select all those options
which are synonyms/antonyms of the word. Select the correct alternative from (1), (2), (3), (4) and (5)
which represents all those synonyms/antonyms.
Q.1. Turbulence
A. commotion B. pandemonium C. fuss D. confusion
(1) Only C (2) Only D (3) Only C and D (4) All except B (5) All of the above
Q.2. Vigilant
A. wary B. pernicious C. circumspect D. captive
(1) Only A (2) Only A and C (3) Only B and D (4) Only B and C (5) All of the above
Q.3. Deficit
A. hierarchy B. Inadequacy C. scantiness D. paucity
(1) Only D (2) Only B and C (3) Only C and D (4) All except A (5) All of the above
Q.4. Fallacious
A. mute B. illogical C. pursue D. shrewd
(1) Only A (2) Only B (3) Only C (4) Only A and B (5) All of the above
Answer
Q.1.(5)
Q.2.(2) For other options:
Pernicious (adj)-hurtful
Captive (n)- prisoner
Q.3.(4)
For other options:
Hierarchy (n)- order
Q.4.(2)
For other options:
Mute (adj)- silent
Shrewd (adj)-clever
---------------------------------------------------------------------------------------------------------------------------------------
--------------------------------------------
CLOZE TEST
Q.1-10.In the following passage there are blanks each of which has been numbered. These numbers
are printed below the passage and against each five words have been suggested, one of which fits
the blanks appropriately. Find out the appropriate word in each case.
The empty ‘Chyawanprash’ containers near the makeshift kitchen at the elephant camp say it all –
that the elephants have been having a healthy, nutritious diet as (1) by the veterinary doctors.
The camp managers say that the elephants get to have the nutritious (2) twice a day – morning before
bath and early evening, walk eight to 10 km on the walking track twice a day and have loads of green
fodder. A few of these elephants also undergo special medication, if necessary.
‘Valli’, a female elephant from the Koodal Azhagar Perumal Koil, Madurai, and ‘Vedanayagi’ another
female elephant from Bhavani, Erode, are two such elephants. Based on the veterinarians’
prescription, the managers are treating the two for foot rot disease.
The elephants get to (3) their legs in a decoction of seven chemicals, a traditional formula, to get over
the problem. The foot rot sets in when the elephants are obese, or stand on hard surface or bitumen-
topped roads for long with very little movement. Likewise, two other elephants are also undergoing
eye treatment to overcome the ‘watery eye’ problem. This occurs when the elephants’ living (4) is hot.
The managers say that the ingredients of the food and the quantity given to the elephants (5) from
one to another and are dependent on the age and gender. Based on the two, a body-mass-index of
sorts is derived and that determines the food and the quantity.
Right at the start of the camp, the managers have noted down the weight of each elephant. This will
be compared to the (6)that they will record when the elephants exit the camp around the second week
of January.
The managers say that one important factor in the camp is giving green fodder, which the
elephants (7) in plenty in the camp. For the weak elephants, the camp managers give twigs of ‘aal’,
‘arasu’ ‘athi’ trees and also ‘koondapanai’. They add that the managers are also (8) the mahouts and
those accompanying the elephants on the ways to keep the animal healthy. This is(9) the mahouts
are with the elephants 24x7. And also because the animal should continue to live in
a (10) environment.
Q.1. (1) assured (2) prescribed (3) decided (4) convinced (5) established
Q.2. (1) calories (2) starvation (3) greenery (4) diet (5) fast
Q.3. (1) immerse (2) sprinkle (3) saturate (4) drown (5) bury
Q.4. (1) envelope (2) appearance (3) effect (4) presence (5) atmosphere
Q.5. (1) dissent (2) turn (3) vary (4) displace (5) remain
Q.6. (1) gravity (2) load (3) substance (4) weight (5) value
Q.7. (1) get (2) capture (3) earn (4) realize (5) access
Q.8. (1) brainwashing (2) developing (3) Educating (4) asking (5) enriching
Q.9. (1) never (2) whatever (3) however (4) since (5) because
Q.10. (1) sound (2) strong (3) fresh (4) healthy (5)safe
ANSWERS
Q.1.(2) Prescribed
For other options:
Convinced (v)- explained
Established (v)- proved
Q.2.(4) Diet
For other options:
Starvation (n)- being hungry
Greenery (n)- green with vegetation
Fast (n)- staying hungry willingly
Q.3.(1) Immerse
For other options:
Sprinkle (v)- spread
Bury
Q. 4.(5) Atmosphere
For other options:
Envelope (n)-cover
Presence (n)-existence
Q.5.(3) Vary
For other options:
Dissent (n)- disagreement
Displace (v)- remove
Remain (v)- be left
Q.6.(4) Weight
For other options:
Gravity (n)-force
Load (n)- burden
Q.7.(1) Get
For other options:
Capture (v)- hold
Access (v)- reach
Q.8.(3) Educating
For other options:
Brainwashing (v)- changing thoughts
Enriching (v)- making better
Q.9.(5) Because
Q10.(4) Healthy
For other options:
Sound (adj)-safe
---------------------------------------------------------------------------------------------------------------------------------------
----------------------------------------------
VOCABULARY
Q.1- 5.In each of the following question four words are given of which two words, are most nearly the
same or opposite in meaning to the given word. Find the two words which are most nearly the same
or opposite in meaning and indicate the number of the correct letter combination, by darkening the
appropriate oval in your answer sheet.
Q.1. Scanty
A. exiguous B. discrete C. meager D. scarce
(1) Only C (2) Only D (3) Only A and D (4) All except B (5) All of the above
Q.2. Accused
A. dilemma B. implicated C. charged D. transient
(1) Only A and C (2) Only B and C (3) Only B andD (4) All except A (5) All of
the above
Q.3. Panic
A. trepidation B. scare C. frenzy D. consternation
(1) Only A (2) Only B (3) Only C (4) Only D (5) All of the
above
Q.4. Elusive
A. tricky B. vigilance C. vandalize D. deceptive
(1) Only B (2) Only D (3) Only A and D (4) Only B and C (5) All of
the above
Q.5. Limit
A. fracas B. bizarre C. restrain D. wanton
(1) Only A (2) Only C (3) All except A (4) All except D (5) All of
the above
ANSWERS
Q.1.(4)
For other options:
Discrete (adj)- different
Q.2.(2)
For other options:
Dilemma (n)- a difficult situation
Transient (adj)-temporary
Q.3.(5)
Q.4.(3)
For other options:
Vandalize (v)- cause destruction
Q.5.(2)
For other options:
Fracas (n)- disturbance
Bizarre (adj)- strange
Wanton (adj)- careless
---------------------------------------------------------------------------------------------------------------------------------------
----------------------------------------------
COMPREHENSION
Q.1-7.Read the following passage carefully and answer the questions given below.
Caste restricts opportunity. Restricted opportunity constricts ability. Constricted ability further restricts
opportunity. Where caste prevails, opportunity and ability are restricted to ever-narrowing circles of
the people. Today, India on the one hand is on the verge of launching its second space shuttle to the
moon; on the other hand, its citizens are 'confusion personified'. Amidst all this chaos about caste,
creed, vote banks, terrorism and much other worldly ridicule, it is time for us to retrospect and take
pragmatic steps towards building a casteless society.
Protesting against conducting a caste-based census would hardly solve the issue, for shunning
ourselves from the truth won't make the bitter truth sweet. A country which has suffered the caste
system for over a thousand years can't become 'casteless' overnight. A caste-based census was
extremely important in a country like ours given the reservations and the quotas that we have to plan.
So, instead of criticizing the system, we need to find a solution within the system which would lead us
to our goal.
The Indian Constitution doesn't give us an option not to belong to any caste. Our society does, but not
the Constitution. An entirely personal and spiritual endeavour such as this definitely doesn't need a
government seal, but the lives and thoughts of the great people who preached secularism failed to
become a movement because of the above mentioned flaw in our Constitution. If an idea gets
constitutionalised or gets governmental back-up, it surely will gain momentum.
It would undoubtedly be a brave decision to enroll oneself as 'casteless', considering the opposition
and criticism these people have to face from their families, and only an intellectually cultured mind can
do that. So be it a 'dalit' or an upper caste person, who elevates himself as 'casteless', because he is
educated, will definitely not need any kind of reservation and he falls into the general category (no
quotas, mind you).
There will be some practical problems. Now, because of this categorical shift, there will be need for
some alterations in the reservation percentage and a few other minor practical issues, to which
solutions can be worked out. But the advantages of this system seem promising.
The children of these 'casteless' parents will, by birth, have nothing tagged to their names, and this
continues, generation to generation. Isn't it wonderful to see a tree pop up amidst acres and acres of
weed? And won't this tree produce more trees? One day, can't we see a thick forest in that place?
A thousand years later, if even this idea gets corrupted, someone else will find a better solution. As of
now, if being casteless can become a constitutional provision, our society will definitely raise itself a
few notches higher.
Q.1. Why has the example of ' a tree popping up amidst acres of weed' been cited by the author?
(1) The best of individuals come from socially backward families
(2) Trees which grow in weed do not survive for long
(3) To convey that a small effort from an individual triggers a big movement
(4) An individual alone cannot do much to change the society
(5) Not clear in the passage
Q.2. What is meant by 'confusion personified' as mentioned in the passage?
(1) Confusion prevails in the matters related to government
(2) Society confuses people with its self made concepts
(3) People remain confused about their own caste
(4) People remain confused about every small or big issue
(5) All of the above
Q.3. Which of the following is the author's concern?
(1) The children of 'casteless parents' will have great problem earning their living
(2) The Indian constitution should provide an option to criticize caste system
(3) The case system is nullifying the effect of the progress India has made in different fields
(4) In future being casteless would mean being 'socially backward'
(5) Not mentioned in the passage
Q.4. Why, according to the passage, a caste-based census inevitable in the Indian context?
(1) India is a nation which offers shelter to people coming from all castes and origins
(2) Due to the provision of reservations and quotas
(3) Because of the superstition and prejudices people do not get rid of.
(4) The politicians exploit emotions of the people to their best advantage
(5) Not mentioned in the passage
Q.5. Which of the following is not true about the person who enrolls himself as 'casteless'?
A. He would be discarded by other members of the society
B. Nothing would be tagged to the names of his children
C. He would not demand reservation
D. His financial independence would be gone as soon as he declares himself 'casteless
(1) Only B (2) Only D (3) Only A and B (4) Only A and D (5) All of the above
Q.6. What notion does Indian constitution hold about 'caste system'?
(1) It is the foundation pillar of many Indian societies
(2) However hard we may try but it cannot be uprooted
(3) It is required to prove one's identity in societal aspect
(4) It is a means for the socially backward people to move ahead in any field
(5) Not mentioned in the passage
Q.7. Which of the following is TRUE in the context of the passage?
(1) Where caste prevails, opportunity and ability are allowed
(2) Our society would get better if the idea of being casteless becomes a constitutional provision
(3) The Indian Constitution provides us an option not to belong to any caste
(4) It is difficult to come up with a better solution to the problem of caste system
(5) All of the above
ANSWERS
Q.1.(3)
Q.2.(4)
Q.3.(3)
Q.4.(2)
Q.5.(4)
Q.6.(5)
Q.7.(2)
---------------------------------------------------------------------------------------------------------------------------------------
----------------------------------------------
ERROR DETECTION
Q.1-5. Read each part of the sentence to find out if there is any error in it. The error, if any, will be in
one part of the sentence. The number of that part is the answer. If there is no error, mark your answer
as (5).
Q.1. A new system to monitor (1)/ the attendance of government school teachers(2)/ is being
introduced (3)/in the district (4)/ No error (5)
Q.2. Labourers are busy applying white cement (1)/on the tombs of the palace (2)/after completions of
which (3)/ they would paint it afresh (4)/ No error (5)
Q.3. The employees are expected (1)/to plan their expenditure (2)/and avail loans (3)/prudently and
responsibly(4)/ No error (5)
Q.4. The poster content provoked (1)/one section of a community (2)/ and the police were deployed
(3)/at the area to control the situation (4)/ No error (5)
Q.5. The international stadium (1/) would offer good facilities (2/)and opportunities to both (3)/national
and international level sportsperson.(4)/ No error (5)
ANSWERS
Q.1.(5)
Q.2.(3) ‘Completion’ would be used – ‘completion’ is an abstract noun which can never have a plural
form
Q.3.(3) ‘Avail’ should be followed by ‘of’
Q.4.(4) ‘In’ should replace ‘at’- to make the sentence correct
Q.5.(5)
---------------------------------------------------------------------------------------------------------------------------------------
----------------------------------------------
SENTENCE IMPROVEMENT
Q.1-5.Which of the phrases (1), (2), (3) and (4) given below should replace the phrase
given in bold in the following sentence to make the sentence grammatically meaningful and correct. If
the sentence is correct as it is and ‘No correction is required’, mark (5) as the answer.
Q.1. He behaved though it was his fault, but we knew he was not responsible for it.
(1) even though it was (2) though it was not
(3) as if it was (4) despite it was not
(5) No correction required
Q.2. She never felt that it was not of her business to get involved in somebody else’s family matter.
(1) were not of her business (2) was none of her business
(3) was of not her business (4) was not of her businesses
(5) No correction required
Q.3. Being born in a certain family is not in our control.
(1) Be born (2) Taking born
(3) By birth (4) Being borned
(5) No correction required
Q.4. I was taken back by his sudden comment on this issue.
(1) would be taken back by (2) was taken backwards by
(3) was taken back for (4) was taken aback by
(5) No correction required
Q.5. In a matter of seconds, we come to know of what is happening anywhere in the world.
(1) came to know of (2) come to be known of
(3) come to know off (4) are coming to know of
(5) No correction required
ANSWERS
Q.1.(3) Use ‘as if it was’ in place of ‘though it was’.
Q.2.(2) Use ‘was none of her business’ in place of ‘was not of her business’.
Q.3.(5)
Q.4.(4) Use ‘was taken aback by’ in place of ‘was taken back by’.
Q.5.(5)
---------------------------------------------------------------------------------------------------------------------------------------
----------------------------------------------
FILL IN THE BLANKS
Q.1-5. In each of the following sentences there are two blank spaces. Below each five pairs of words
have been denoted by numbers (1), (2), (3), (4) and (5). Find out which pair of words can be filled up
in the blanks in the sentences in the same sequence to make the sentence meaningfully complete.
Q.1. The athletes have been able to establish how the ——————substance——————— their
body.
(1) prohibit, enters (2) prohibits, enter (3) prohibiting, entering (4) prohibit, enter (5)
prohibited , entered
Q.2. The judges made it clear that they were not —————to pass —————direction.
(1) apprehension, any (2) divided , many (3) inclined , any (4) ruled , some (5)
curiosity , much
Q.3. HRD ministry issued a detailed gazette notification ——————all the central educational
institutions to abide ———the decision.
(1) asking, by (2) demanded, from (3) revert, to (4) applaud, at (5) giving , in
Q.4. They have been requesting the authority for —————sometime, but no ——————response
has come through.
(1) quiet, desire (2) quite , positive (3) quiet , authority (4) quit, negativity (5) quite ,
potential
Q.5. ———————yarn available at a low price had been one of the —————demands of the
weavers.
(1) Doing, measure (2) Implementing, best (3) Happening , popularity (4) Making , major
(5) Associating , most
ANSWERS
Q.1.(5) prohibited , entered
Q.2.(3) Inclined , any
For other option:
Apprehension- fear
Q.3.(1) asking , by
For other options:
revert- to reply
applaud- to express approval
Q.4.(2) quite , positive
For other options:
Authority – person having power
Potential - having strength
Q.5.(4) Making , major
For other options:
implementing – to put to practice
associating- connecting
---------------------------------------------------------------------------------------------------------------------------------------
----------------------------------------------
REARRANGEMENT
Q.1-5. Rearrange the following six sentences (A), (B), (C),(D) and (E) in the proper sequence to form
a meaningful paragraph; then answer the questions given below them.
(A) Without a productive labour force.
(B) Among the natural resources which can be called upon in natural plans for development.
(C) Including effective leadership and intelligent middle management.
(D) Possibly the most important is human labour.
(E) No amount of foreign assistance or of natural wealth can ensure successful development.
Q.1. Which of the following should be the SECOND sentence after rearrangement?
(1) A (2) B (3) C (4) D (5) E
Q.2. Which of the following should be the FOURTH sentence after rearrangement?
(1) A (2) B (3) C (4) D (5) E
Q.3. Which of the following should be the FIRST sentence after rearrangement?
(1) A (2) B (3) C (4) D (5) E
Q.4. Which of the following should be the Last sentence after rearrangement?
(1) A (2) B (3) C (4) D (5) E
Q.5. Which of the following should be the THIRD sentence after rearrangement?
(1) A (2) B (3) C (4) D (5) E
ANSWERS
B is the first sentence after rearrangement because only B is independent .B and D is mandatory pair
A continues the idea given in B and D so it is 3rd sentence and A,C is mandatory pair, and E ends the
topic so it is last.
Q.1.(2) B
Q.2.(4) D
Q.3.(1) A
Q.4.(3) C
Q.5.(5) E
The correct sequence is BDACE
---------------------------------------------------------------------------------------------------------------------------------------
----------------------------------------------
MISAPPROPRIATION
Q.1-5. In each question below a sentence with four words printed in bold type is given. These are
numbered (1),(2),(3) and (4) . One of these four words printed in bold may be either wrongly spelt or
inappropriate in context of the sentence. Find out the word which is wrongly spelt or inappropriate if
any. The number of that word is your answer. If all the words printed in bold are correctly spelt & also
appropriate in the context of the sentence, mark (5) “All correct” as your answer.
Q.1. The Economic upheaval in one part of the world has direct bearing on the economic activties
(1) (2) (3) (4)
across the globe. All correct.
(5)
Q.2. All parts of the world are tied to each other inextricable through electronic media. All correct
(1) (2) (3) (4) (5)
Q.3. At the summit, extensive discussions were held on the challenges currently being faced by
the
(1) (2) (3)
human society which include debt and sustainabilyty. All correct
(4) (5)
Q.4. The world leaders also shed light on various other partinant issues facing the world at present.
(1) (2) (3)
(4)
All correct
(5)
Q.5. Renuable energy technologies have made significant contributions to solving some of the
most (1) (2) (3)
urgent problems the world is facing. All correct
(4) (5)
ANSWERS
Q.1.(4) Activities in the correct spelling
Q.2.(3) Inextricably should be used
Q.3.(4) Sustainability is the correct spelling
Q.4.(3) Pertinent is the correct spelling
Q.5.(1) Reviewable is the correct spelling
---------------------------------------------------------------------------------------------------------------------------------------
----------------------------------------------
SENTENCE IMPROVEMENT
Q.1-5. In each question below a sentence with four words printed in bold type is given. These are
numbered (1),(2),(3) and (4) . One of these four words printed in bold may be either wrongly spelt or
inappropriate in context of the sentence. Find out the word which is wrongly spelt or inappropriate if
any. The number of that word is your answer. If all the words printed in bold are correctly spelt & also
appropriate in the context of the sentence, mark (5) “No error” as your answer.
Q.1. The red blossoms accented the prevailing pink of the house front. No error
(1) (2) (3) (4) (5)
Q.2. Our strongest reason for siding with the communist is because our father is a communist.
(1) (2) (3) (4)
No error
(5)
Q.3. He ordered the servants to prepare a feast had the gates opened and to fire a salute.
(1) (2) (3) (4)
No error
(5)
Q.4. The pilot revealed that his airbus was carrying not less than three hundred passengers
(1) (2) (3)
last evening. No error
(4) (5)
Q.5. The nineteenth Ammendement to the constitution gave women the right to vote in the
(1) (2) (3)
elections of 1920. No error
(4) (5)
ANSWERS
Q.1.(2) Use ascented in place of accented.
Q.2.(4) Use ‘that’ in place of because as the word ‘reason’ has already been used.
Q.3.(3) Use “when” in place of had
Q.4.(3) Write more than in place of less than
Q.5.(1) The correct spelling is amendment.
---------------------------------------------------------------------------------------------------------------------------------------
----------------------------------------------
ERROR DETECTION
Q.1-5. Read each sentence to find out if there is any error in it. The error, if any, will be in one part of
the sentence. The number of that part is the answer. If there is no error, the answer is (5).
Q.1. While one part of the TV programme carried the football game ,the (1)/ other part shows the (2)/
training(3)/ of the tennis(4)/ No Error(5)
Q.2. Prior to the examination(1) / the instructor had told us to keep (2)/ an eye on the time to(3)/ write
clearly and no cheating(4) /No Error (5)
Q.3. The rainy season(1)/ had already began(2)/ by the time they arrived(3)/ in Burma that year(4)/
No Error (5)
Q.4. Whenever these three (1)/ eminent scientist spoke to(2)/ each other an argument(3)/ was sure(4)
No Error (5)
Q.5. There was scarcely(1)/ on time given to think about(2)/the problem before the bell (3)/at the end
of class(4)/ No Error(5)
ANSWERS
Q.1.(3) "Showed" should replace "shows because the sentence is in past tense."
Q.2.(2) "Not to cheat" should replace "nocheating because and connects the safe same from of
verb."
Q.3.(2) "Begun"should replace "begun"because had is followed by v3 from.
Q.4.(3) "One another" should replace"each other" because one another is used for more than two
persons.
Q.5.(2) "Any" should replace "no" because scascely already given negative sense and double
negative is not used in a sentence.
---------------------------------------------------------------------------------------------------------------------------------------
----------------------------------------------
FILL IN THE BLANKS
Q.1-5. In each of the following sentences there are two blank spaces. Below each five pairs of words
have been denoted by numbers (1), (2), (3), (4) and (5). Find out which pair of words can be filled in
the blanks inorder to make a meaningful sentence.
Q.1. Football evokes a ________ response in India compared to cricket, that almost _____the nation.
(1) magnifying, manipulates (2) fascinating, forbids (3) lukewarm, electrifies (4) wild,
shakes (5) tentative, guides
Q.2. The cigarette manufactures’ association has _________ to _________in aggressive promotion.
(1) decided, indulge (2) calculated, call (3) forgotten, motivate (4) predicted, plant (5)
cared, sell
Q.3. Social studies, Science, ______of health and safety, the very atmosphere of classroom, are few
of the important areas for the ______of proper emotional reactions.
(1) course, clearance (2) matters, formation (3) areas, rulings (4) radius, spirit (5) mature,
provision
Q.4. The chief idea of one very ________type of traveler is to see as many objects of ________ as he
possibly can.
(1) measure, hate (2) minor, power (3) error, fun (4) yielding, utility (5) common, interest
Q.5. Learning is more efficient when it is _______. It is less efficient when it is _____.
(1) fast, slowly (2) rapidly, turtle-slow (3) tedious, obedient (4) fun, drudgery (5) growing,
imparting
ANSWERS
Q.1.(3) Lukewarm- indifferent
Electrifies- thrill, amaze
For other options:
Magnifying- enlarging, intensifying
Manipulates- maneuver
Fascinating- interesting, spellbinding
Forbid- prohibit an action
Tentative- conditional, experimental
Q.2.(1) decided, indulge
Indulge- perform service
Predicted- express an outcome in advance
Q.3.(2) matters, formation
Q.4.(5) common. Interest
For other options:
Yielding- accommodating
Q.5.(4) fun, Drudgery
Drudgery- hard, tedious work
For other options:
Turtle-slow- very slow
---------------------------------------------------------------------------------------------------------------------------------------
----------------------------------------------
COMPREHENSION
Q.1-10.Read the following passage carefully and answer the questions given below it. Certain words
have been printed inbold to help you locate them while answering some questions.
The Reserve Bank of India (RBI) stipulated that debit cards would be issued to customers having
Savings Bank and Current Accounts but not to cash credit or loan account holders. Banks may issue
only online debit cards, including co-branded debit cards where there is an immediate debit to the
customers’ account, and where straight through processing is involved.
Banks are, henceforth, not permitted to issue offline-debit cards. Banks which are now issuing offline
debit cards may conduct a review of their offline debit card operations and discontinue operations of
such cards within a period of six months from the date of the circular issued.
Banks were also asked to ensure that customers were duly informed regarding switching over to
online debit cards. However, till such time as offline cards were phased out, the outstanding balances
/ unspent balances stored on the cards would be subject to computation of reserve requirements.
As per the notification, no bank shall dispatch a card to a customer unsolicited, except in the case
where the card is a replacement for a card already held by the customer. Each bank shall make
available to the cardholders in writing, a set of contractual terms and conditions governing the issue
and use of such a card. These terms shall maintain a fair balance between the interests of the parties
concerned and shall be expressed clearly.
The terms shall specify that the bank shall be responsible for direct losses incurred by a cardholder
due to a system malfunction directly within the bank’s control. However, banks would not be
held liable for any loss caused by a technical breakdown of the payment system if the breakdown of
the system was recognizable for the cardholder by a message on the display of the device or
otherwise known. The responsibility of the bank for the non-execution or defective execution of the
transaction is limited to the principal sum and the loss of interest subject to the provisions of the law
governing the terms.
With a view to reducing the instances of misuse of lost/stolen cards, the apex bank asked banks to
issue cards with photographs of the cardholder or any other advanced methods that may evolve from
time to time. Banks should undertake review of their operations/issue of debit cards on half-yearly
basis. The review would include, inter-alia, card usage analysis, including cards not used for long
durations due to their inherent risks.
Banks were asked to ensure to put in place an effective mechanism for redressal of customer
complaints. The name, designation, address and contact number of important executives as well as
the grievance redressal officer of the bank may be displayed on the website. Also, there should be a
system of acknowledging customers’ complaints for follow-up, such as complaint number / docket
number.
Q.1. Why have banks been asked to discontinue their offline debit card operations?
(1) Offline debit card issuing system has been facing problems for quite sometime
(2) As per the new circular, debit cards will be issued online only
(3) Banks should make use of the latest technology available
(4) With offline debit card system any transaction takes a long time.
(5) None of these
Q.2. Which of the following has been referred as ‘Apex bank’ in the passage?
(1) Bank Issuing the online debit card
(2) Bank which still offers offline debit cards
(3) Every bank which is governed by RBI
(4) Reserve Bank Of India
(5) None of these
Q.3. Which of the following is/are essential to be displayed on website for redressal of customer
complaints?
A. Complaint number / docket number
B. Complaint type
C. Details of grievance redressal officer of the bank
D. Details of important executives
(1) Only C (2) Only A and C (3) All except B (4) All except A (5) All of the above
Q.4. How much time have the banks been given to discontinue operations of offline debit cards ?
(1) 1 year (2) 3 months (3) 1 month (4) 6 months (5) 2 weeks
Q.5. Which of the following is FALSE, according to the passage?
(1) Every year Banks should undertake review of their operations/issue of debit cards.
(2) Banks should resort to advanced methods to reduce the instances of misuse of lost/stolen cards
(3) Bank would be responsible for direct losses incurred by a cardholder because of a system
malfunction directly within the bank’s control
(4) Each bank should make available to the cardholders in writing, a set of contractual terms and
conditions governing the issue and use of a debit card
(5) All of the above
Q.6. Which of the following should maintain a fair balance between the interests of the parties
concerned, as per the notification?
(1) Contractual terms
(2) Circular issued
(3) Review of banks’ operations/issue of debit cards
(4) System of acknowledging customers’ complaints
(5) Not mentioned in the passage
Q.7-8.Choose the word most similar in meaning to the word printed in bold, as used in the passage.
Q.7. Liable
(1) Responsible (2) Innocent (3) Caught (4) Inclined (5) Disciplined
Q.8. Duly
(1) Suitably (2) Equally (3) Nicely (4) Properly (5) Liberally
Q.9-10.Choose the word which is most nearly the OPPOSITE in meaning as the word printed in bold
as used in the passage.
Q.9. Inherent
(1) Integral (2) External (3) Essential (4) Incidental (5) Original
Q.10. Dispatch
(1) Hurry (2) Send (3) Receive (4) Perform (5) Destroy
ANSWER
Q.1.(2)
Q.2.(4)
Q.3.(3)
Q.4.(4)
Q.5.(1)
Q.6.(1)
Q.7.(1) Responsible
For other options:
Inclined (v)- interested
Q.8.(4) Properly
For other options:
Liberally (adv)- with liberty
Q.9.(2) External
For other options:
Essential (adj)- important
Incidental (adj)- by chance
Q.10.(3) Receive
---------------------------------------------------------------------------------------------------------------------------------------
----------------------------------------------
REARRANGEMENT
Q.1-5. Rearrange the following seven sentences (A), (B), (C), (D), (E), (F) and (G) in the proper
sequence to form a meaningful paragraph; then answer the questions given below.
(A) But seriously, how much would you pay to know what thoughts are swimming around in someone
else’s head?
(B) In most fictional movies, thus, the idea of reading minds-of seeing the private intentions of
another, and the possibility of intervening in those plans - has always been highly attractive.
(C) Such fantastic questions have long been the bread and butter of fiction,
(D) Today, more than four centuries since the phrase, “A penny for your thoughts?”, was first
recorded, inflationary accounting makes that ancient penny was worth more than $40.
(E) The going rate for a “thought”-a probe into the thinking of another-was once quite a bargain.
(F) And if you could really know their truthfulness, how much more would you pay?
(G) Even with the sliding value of the dollar, this still seems quite a bargain.
Q.1. Which of the following should be the SECOND sentence after rearrangement?
(1) F (2) D (3) C (4) G (5) B
Q.2. Which of the following should be the FOURTH sentence after rearrangement?
(1) A (2) B (3) G (4) D (5) F
Q.3. Which of the following should be the SIXTH sentence after rearrangement?
(1) F (2) G (3) C (4) B (5) A
Q.4. Which of the following should be the SEVENTH (Last) sentence after rearrangement?
(1) A (2) B (3) C (4) D (5) E
Q.5. Which of the following should be the FIRST sentence after rearrangement?
(1) A (2) B (3) C (4) D (5) E
ANSWERS
Q.1-5. The correct rearrangement is EDGAFCB
Q.1.(2) D
Q.2.(1) A
Q.3.(3) C
Q.4.(2) B
Q.5.(5) E
---------------------------------------------------------------------------------------------------------------------------------------
----------------------------------------------
ERROR DETECTION
Q.1-5. Read each part of the sentence to find out if there is any error in it. The error, if any, will be in
one part of the sentence. The number of that part is the answer. If there is no error, mark your answer
as (5).
Q.1. Medicines for treatment of tuberculosis (1)/will now be available (2)/for free at all (3)/chemical
shops and corporate hospitals. (4)/ No error (5)
Q.2. A young community activist began a road journey (1)/though 21 countries to emphasize(2)/ the
need for peace (3)/ between India and Pakistan (4)/ No error (5)
Q.3. Urdu language played (1)/a pivotal role (2)/ in national integration (3)/during the war of
independence. (4)/ No error (5)
Q.4. Without waiting for(1)/ the instructions from the government(2)/ some cable operators in (3)/ the
city had block the news. (4)/ No error (5)
Q.5. A long coastline means(1)/ better opportunities for (2)/ fishing which solves unemployed (3)/ and
food problems to a certain extent (4)/ No error (5)
ANSWERS
Q.1.(5)
Q.2.(2) ‘Through’ should replace ‘to’ to make the sentence meaningful
Q.3.(5)
Q.4.(4) use ‘blocked’ in place of ‘block’.
Q.5.(3) use ‘unemployment’ instead of ‘unemployed’.
---------------------------------------------------------------------------------------------------------------------------------------
----------------------------------------------
HOMONYMS
Q.1-5.In each question, there are three sentences. Each sentence has pairs of words/phrases that are
italicized and highlighted. From the italicized and highlighted word(s)/ phrase(s), select the most
appropriate word(s)/ phrase(s) to form correct sentence. Then, from the options given, choose the
best one.
Q.1. Characters tie events (A)/ invents (B) in a story together and provide a thread of continuity and
meaning.
His close confidant (A)/ confident (B) knew all of his secrets and was not ready to help us.
Though (A)/ Through (B) known for its artistic merits, the film nonetheless did poorly at the box office.
(1) AAA (2) BBB (3) ABA (4) BAB (5) AAB
Q.2. The meeting ended without any plans for farther (A)/ further (B) discussions.
As a result, everyone treats the other as an adversely (A)/ adversary (B) rather than an ally.
He studied the creation, inhabitation (A)/ inhibition (B) and demise of the colony.
(1) AAA (2) BBB (3) BBA (4) AAB (5) BAB
Q.3. She prepared an apple pie for the desert (A)/ dessert (B) today.
Avocado pear (A)/ peer (B) trees usually require sub tropical conditions to flourish.
Amount of commission earned will vary (A)/ very (B) between employees.
(1) AAB (2) BAA (3) ABA (4) BAB (5) AAA
Q.4. Beside (A)/ Besides (B) being difficult, grammar can be confusing.
There are hundreds of children around the country who will be adversely affected
(A)/ effected (B) by this ruling.
The balance of power will shift to the East as China and India involve (A)/ evolve (B).
(1) BAB (2) ABA (3) AAB (4) BBA (5) BAA
Q.5. It was a tough situation and Manasi was taking pain (A)/ pane (B) to make it better.
The police, during the raid, ceased (A)/ seized (B) all the arms and ammunitions in
their possession.
The principal (A)/ principle (B) scolded the students for creating chaos during the recess.
(1) AAA (2) BBB (3) BAB (4) AAB (5) ABA
ANSWERS
Q.1.(1) AAA
Q.2.(3) BBA
Q.3.(2) BAA
Q.4.(1) BAB
Q.5.(5) ABA
---------------------------------------------------------------------------------------------------------------------------------------
--------------------------------------------
CLOZE TEST
Q.1-10.In the following passage there are blanks each of which has been numbered. These numbers are
printed below the passage and against each five words have been suggested, one of which fits the
blanks appropriately. Find out the appropriate word in each case.
Social networking sites are immensely useful in (1) boundaries when spreading social messages.
But the flip (2)of social networking sites is their (3) for spreading destruction and social malaise. The
latter trend was seen in two recent uses of social networking. The first one was of a man selling his
grandchild to a businessman in Delhi for Rs 8 lakh after striking a deal using Facebook. The other
was the Dow Jones falling by 143 points after hackers sent a message — ‘Two Explosions in the
White House and Barack Obama is injured’ — from the Twitter handle of a news organization. The
Syrian Electronic Army, which had (4) the Reuters feed last year, took credit for this. Social
networking sites have shrunk the world and connected the most disparate of people like never before.
However, it is the challenges (5) by its dark underbelly that governments around the world have not
been able to control.
China, despite it denying any part in it, has been (6) in its cyber attacks on countries and
organizations that have spoken against its interests. A recent example of Beijing’s use of cyber space
to (7) unpalatable views was the attack on the New York Times’ website after it ran a feature exposing
former premier Wen Jiabao’s wealth. Many countries, including India, have been victims of Chinese
hackers. Al-Qaeda has been using the Internet, especially video-sharing platforms and social
networking sites, for (8) and recruitment.
To think that a single tweet can affect international markets and in other cases escalate tension
between two nations is spine chilling. Does this mean that the government should roll down the
shutters on social networking sites? Definitely not. Real-time censoring of Internet traffic, given the
volume (which is only going to increase), is unthinkable. Governments, especially in India, should,
through its various wings, create more Internet awareness and give, cyber monitoring cells in the
police and other agencies ,more teeth. Internet and social networking sites are here to stay and
evolve; shying away from them or banning them will be (9). Governments should be one step ahead
of forces that exploit these platforms for (10) purposes. This is the only way it can really get a handle
on things.
Q.1. (1) migrating (2) replicating (3) transcending (4)accomplishing (5) organizing
Q.2. (1) turn (2) schedule (3) cases (4) corner (5) side
Q.3. (1) potential (2) inherent (3) sluggish (4) latency (5) litigation
Q.4. (1) concerned (2) hacked (3) captivated (4) overwrought (5) irate
Q.5. (1) masquerade (2) preferred (3) juxtaposed (4) posed (5) transformed
Q.6. (1) notorious (2) obvious (3) adorable (4) chosen (5) salient
Q.7. (1) utilize (2) welcome (3) assist (4) hoodwink (5) stymie
Q.8. (1) publication (2) brainstorming (3) propaganda (4) fundamental (5) tradition
Q.9. (1) intellectua (2) counterproductive (3) miserably (4) idle (5) vainglorious
Q.10. (1) ethical (2) voluminous (3) lawful (4) virtuous (5) nefarious
ANSWERS
Q.1.(3) Transcending
For other options:
Migrating (v)-traveling to another place
Replicating (v)- copying
Accomplishing (v)- completing
Q.2.(5) Side
Q.3.(1) Potential
For other options:
Inherent (adj)-elementary
Sluggish (adj)-inactive
Latency (n)-suspension
Litigation (n)-matter coming before court
Q.4.(2) Hacked
For other options:
Overwrought (adj)- exhausted and excited
Irate (adj)- angry
Q.5.(4) Posed
For other options:
Masquerade (n)-disguise
Juxtaposed (v)- place side by side
Q.6.(1) Notorious
For other options:
Obvious (adj)- clear
Adorable (adj)-lovable
Salient (adj)- prominent
Q.7.(5) Stymie
For other options:
Hoodwink (v)- deceive
Q.8.(3) Propaganda
For other options:
Fundamental (adj)- basic
Q.9.(2) Counterproductive
For other options:
Miserably (adv)- badly
Vainglorious (adj)-boastful
Q.10.(5) Nefarious
For other options:
Voluminous (adj)-vast
Virtuous (adj)-ethical
---------------------------------------------------------------------------------------------------------------------------------------
--------------------------------------------
FILL IN THE BLANKS
Q.1-5.In each of the following sentences there are two blank spaces. Below each five pairs of words have been denoted
by numbers (1), (2), (3), (4) and (5). Find out which pair of words can be filled in the blanks inorder to make a
meaningful sentence.
Q.1. A day —— call is one which tells you to buy or ————— a share on the same day.
(1) trading, share (2) trading, sell (3) trades, sell (4) trading, buy (5) trading, close
Q.2. It is ———— to read the SMS: “Make money trading equity derivatives up to 95% —————.
(1) heartening, accurate (2) hard, only
(3) heartening, approximation (4) hard, accurate
(5) heartening, accuracy
Q.3. The Food Corporation of India is —— to sell wheat ———— spot exchanges for bulk and retail purchasers.
(1) making, through (2) planning, while (3) planning, through
(4) making, across (5) making, within
Q.4. The post - reforms ———— economic sentiment has —————— up pre-placement offers and pre-placement
interviews at the country’s top non-IIM management institutes.
(1) negative, fired (2) positive, fired (3) positive, shot
(4) positive, given (5) positive, done
Q.5. When the mid-term review of the employee’s performance does not ————— expectations, an escalation to
senior managers may not help because often, senior managers are ——— in reviews.
(1) meet, involve (2) work, indulged (3) meet, indulge
(4) beyond, involved (5) meet, involved
ANSWERS
Q.1.(2) trading, sell
Q.2.(5) heartening, accuracy
For other options:
Approximation (n)-estimation
Q.3.(3) planning, through
For other options:
Across (prep)-in all parts of
Through (prep)- by the medium of
Q.4.(2) positive, fired
For other options:
Shot (v)- went up/ incresed
Q.5.(5) Meet, involved
For other options:
Beyond (prep)- Over
Indulge (v)- get into something
---------------------------------------------------------------------------------------------------------------------------------------
---------------------------------------
REARRANGEMENT OF SENTENCES
Q.1-5.Rearrange the following sentences (A),(B),(C), (D),(E),(F),(G) and (H) in the proper sequence to form a meaningful
paragraph, then answer the questions given below them.
(A) As one who had the good fortune of being in the company
(B) The world of Indian classical music has become poorer
(C) He was very kind at heart and would pacify them immediately
(D) Of some of his disciples in the 1960s, I was wonderstruck by the preparation
(E) He expected from his students before a concert.
(F) We have lost a musician who could hold the audience spellbound
(G) With the passing of the great sitar maestro, Pandit Ravi Shankar.
(H) While he used to get annoyed with them at the slightest sign of imperfection
Q.1. Which of the following is the SIXTH statement after rearrangement?
(1) F (2) D (3) H (4) A (5) E
Q.2. Which of the following is the FIRST statement after rearrangement?
(1) E (2) D (3) H (4) B (5) A
Q.3. Which of the following is the EIGHTH statement after rearrangement?
(1) F (2) C (3) E (4) A (5) G
Q.4. Which of the following is the SECOND statement after rearrangement?
(1) G (2) D (3) C (4) E (5) B
Q.5. Which of the following is the THIRD statement after rearrangement?
(1) C (2) D (3) F (4) A (5) E
ANSWERS
Q.1.(5)
Q.2.(4)
Q.3.(2)
Q.4.(1)
Q.5.(3)
The arrangement is BGFADEHC. The paragraph is about Pandit Ravi Shankar and the
irreparable loss caused to the music world due to his death. Further, it has been mentioned he was a
perfectionist and had the potential to charm the audience.
---------------------------------------------------------------------------------------------------------------------------------------
----------------------------------
MISAPPROPRIATION
Q.1-8.In each set below five words have been printed which are numbered (1), (2), (3),(4) and (5). One of
these words may be wrongly spelt.Find out the wrongly spelt word. The number of that word is the
answer. If all the words are correctly spelt and are appropriate, the answer is (5) i.e. "All correct".
Q.1. Inadequate employ skills and lack of confidence are some of the major hindrances these
(1) (2) (3)
children face in finding gainful employment All correct
(4) (5)
Q.2. The professor aprised the students about the scope of psychology and its role and functions in
(1) (2) (3)
our daily routine All correct
(4) (5)
Q.3. The real estate businessmen have created an artificial scarcity of land thereby making it
(1) (2) (3)
costly All correct
(4) (5)
Q.4. One of the biggest concerns for pet owners is the constraint that they cannot take their pets
(1) (2) (3)
out when they roam round in public places All correct
(4) (5)
Q.5. With stock markets on the upswing, iquity mutual funds have started to reward investors
(1) (2) (3) (4)
All correct
(5)
Q.6. All the elected representatives in the state would soon be able to availed the benefits of
(1) (2)
cashless treatment at selected government and private hospitals All correct
(3) (4) (5)
Q.7. An increasing number of accidents happen as cranes are driven along roads from
(1) (2)
one worksight to another even during peak hours All correct
(3) (4) (5)
Q.8. Weeding out fake ration cards is going on and once the work is completed,issuance of new
(1) (2) (3) (4)
cards would begin. All correct
(5)
ANSWERS
Q.1.(1) ‘Employable ‘ should be used- an adjective is required at this place
Q.2.(1) ‘Apprised’ is the correct spelling
Q.3.(5)
Q.4.(4) ‘Roaming around’ is used – which means ‘walking around for enjoyment’
Q.5.(3) ‘Equity’ is the correct spelling
Q.6.(2) ‘Avail’ should be used – first form of verb should come after ‘to’
Q.7.(3) ‘Worksite’ is the correct word which means ‘site where the work is in progress’
Q.8.(5)
---------------------------------------------------------------------------------------------------------------------------------------
---------------------------------------
COMPREHENSION
Q.1-10.Read the following passage carefully and answer the questions given below it. Certain words have
been printed inbold to help you locate them while answering some questions.
During the last two decades, physicians and mental health professionals have begun to discover
the limitationsof western allopathic medicine. The focus is on pathology and disease rather than on
prevention. The destructiveness of so many pharmaceutical and surgical remedies, the separation of
physical and emotional problems, theassumption of an asymmetrical relationship between an all-
powerful physician and a submissive patient have led clinics and researchers to look for answers in
other traditions and cultures.
It is indeed disappointing in this context, that in India, with its long tradition of the body-mind
continuum, health efforts are so largely dependent on the allopathic system which is now being
increasingly rejected by western medical scientists. However, any wide open, fuzzy field like ‘holistic
health’ offers abundant opportunities for fraud and overpromise. The new paradigm of health is not a
licence for quackery. Holistic health is a perspective, not a specialisation or a discipline.
The body-mind is a process, so is healing or making whole. It was discovered that it is not the
simple physical change but rather the state of mind that is the key to health. This state has been
called ‘restful alertness,’ ‘passive volition’, ‘deliberate betting’ etc. Cumulative stresses seem to melt
under this seemingly paradoxical state, restoringnatural flow to the body-mind whirlpool.
Clearly, health services, as we know them, are essential but, equally clearly, they are not enough.
There is an urgent need to blend the ancient traditional ways with some of the discoveries of modern
medicine to make way for a truly healthy society. The first and most important area deserving urgent
attention is child health care.
Conventionally, it is the doctor and the hospitals that are seen at the centre of all health care; the
mother and the child are at the periphery. It is perhaps necessary to see the mother as the centre of
child health care. It is she who is the highest level health worker not by training or in qualifications but
in time and love, in the special knowledge of her own children, in the breadth of ‘integrated services’
she provides and in the permanent presence she brings to her child’s life. This indeed was the basis
of the ancient child care practice and it is well that it is being rediscovered. The implications of this are
manifold. Mother must be imparted the knowledge, skills and techniques required for her to do the
job well. The old civilisations did this through word of mouth from one generation to another. Modern
civilisations can improve on this but the essence will remain the same.
Q.1. Which of the following is most likely in disagreement with the passage?
(1) The western allopathic system of cure places less emphasis on prevention of disease and more on
its treatment.
(2) The western allopathic system is an extension of India’s ancient medical system.
(3) The allopathic system is being rejected by Western medical scientists.
(4) The allopathic system has takers in India.
(5) All of the above
Q.2. Which one of the following does not characterize the ‘holistic’ approach in medicine?
(1) It is as susceptible to fraudulent practices as is any other approach.
(2) Holistic health is a perspective and not a specialization or a discipline.
(3) It is in tune with western culture and traditions.
(4) None of the above
(5) All of the above
Q.3. Which of the following is in agreement with the passage?
(1) A judicious blending of traditional and modern systems of medicine is what is required.
(2) More research work needs to be done and facilities increased for western allopathic medicine to
be successful.
(3) The centre for child health-care is the primary health-care centre
(4) The allopathic system of cure is increasingly being rejected by Indians
(5) Both 3&4
Q.4. The centre of child health care should be
(1) father (2) mother (3) doctor (4) hospitals (5) medical fraternity
Q.5-7.Choose the word most similar in meaning to the word printed in bold, as used in the passage.
Q.5. Limitations
(1) Prohibition (2) Benefits (3) Constraints (4) Extent (5) Infinity
Q.6. Assumption
(1) Manipulation (2) Succession (3) Hindrance (4) Belief (5) Deduction
Q.7. Imparted
(1) Separated (2) United (3) Dragged (4) Pushed (5) Given
Q.8-10. Choose the word which is most nearly the OPPOSITE in meaning as the word printed in bold as used
in the passage.
Q.8. Rejected
(1) Declined (2) Supported (3) Insulted (4) Abandoned (5) Deserted
Q.9. Quackery
(1) Mischief (2) Honesty (3) Liberty (4) Sound (5) Dissatisfaction
Q.10. Restoring
(1) Saving (2) Accumulating (3) Adopting (4) Hampering (5) Improving
ANSWERS
Q.1.(2) The passage dwells on the weakness of the western allopathic system of cure. The emphasis is more
on the pathology and disease than on prevention, so much so that the system is increasingly being
rejected by western medical scientists.
Q.2.(3)
Q.3.(1)
Q.4.(2)
Q.5.(3) Constraints
Limitations- disadvantages
Q.6.(4)
Q.7.(5) Given
Q.8.(2) Supported
Q.9.(2) Quackery- Deceitfulness, deception
Q.10.(4) Hampering
----------------------------------------------------------------------------------------------------------------
-------------------------------------------------------------------
PARAGRAPH COMPLETION
Q.1-5.In each of the following paragraphs there is one blank space. Below each sentence has been
denoted by numbers (1), (2), (3), (4) and (5). Find out which sentence can be filled up in the blank to
make the paragraph meaningfully complete.
Q.1. It is difficult being a marketer in the digital age but now you feel you are on top of things. Your
corporate website is finally optimised. ——————————————and have successfully grown
communities on all of them. For those segments of your business that require mobile apps, you’ve
developed apps straddling various operating systems. You’ve got the hang of digital out-of-home and
have started building interactivity into your retail spaces as well. You have tied up two sets of
agencies for your digital and physical strategies and are happy you roped in the absolute best in the
business.
(1) Customers today are bouncing in and out of multiple channels across different devices.
(2) You have leveraged most of the social channels your customers are likely to use
(3) Corporate budgets have yet to align to support all these channels
(4) It is increasingly difficult to determine where in-store retailing ends and e-commerce begins.
(5) Companies need to build an integrated communication strategy.
Q.2. ——————————————now as the Indian Railways is working on a brand new ticketing
website that promises to be three times faster than the existing IRCTC ticketing portal. The larger plan
is to phase out the present IRCTC site. And if all goes well, the new portal would start functioning by
November. In a step towards setting up a new portal, its Beta version (development stage) would be
available on the www.irctc.co.in The users wanting a new booking experience different from the
present IRCTC site can access it.
(1) From the existing 2000 tickets per minute, the idea is to create a system
(2) The aim is to achieve user delight through technology solution in railways
(3) IRCTC books five lakh tickets per day
(4) A separate journey planning would be available from the ticket booking stage so as to ease out the
booking process.
(5) Failed transactions during online booking of train tickets would be a thing of the past
English
English
English
English
English
English
English
English
English
English
English
English
English
English
English
English
English
English
English
English
English
English
English
English
English
English
English
English
English
English
English
English
English

More Related Content

Similar to English

I N C O M E T A X L A W O F I N D I A
I N C O M E  T A X  L A W  O F  I N D I AI N C O M E  T A X  L A W  O F  I N D I A
I N C O M E T A X L A W O F I N D I A
Dr. Trilok Kumar Jain
 
Industrial Development & Regulation Act & Other Business Laws
Industrial Development & Regulation Act & Other Business LawsIndustrial Development & Regulation Act & Other Business Laws
Industrial Development & Regulation Act & Other Business Laws
Dr. Trilok Kumar Jain
 
` H O W T O W I N D U P A S I C K C O M P A N Y
`  H O W  T O  W I N D  U P  A  S I C K  C O M P A N Y`  H O W  T O  W I N D  U P  A  S I C K  C O M P A N Y
` H O W T O W I N D U P A S I C K C O M P A N Y
Dr. Trilok Kumar Jain
 
C:\Fakepath\Tr Epowerpoint
C:\Fakepath\Tr EpowerpointC:\Fakepath\Tr Epowerpoint
C:\Fakepath\Tr Epowerpoint
Alfredaemonya
 

Similar to English (20)

I N C O M E T A X L A W O F I N D I A
I N C O M E  T A X  L A W  O F  I N D I AI N C O M E  T A X  L A W  O F  I N D I A
I N C O M E T A X L A W O F I N D I A
 
Industrial Development & Regulation Act & Other Business Laws
Industrial Development & Regulation Act & Other Business LawsIndustrial Development & Regulation Act & Other Business Laws
Industrial Development & Regulation Act & Other Business Laws
 
` H O W T O W I N D U P A S I C K C O M P A N Y
`  H O W  T O  W I N D  U P  A  S I C K  C O M P A N Y`  H O W  T O  W I N D  U P  A  S I C K  C O M P A N Y
` H O W T O W I N D U P A S I C K C O M P A N Y
 
Income tax law of india
Income tax law of india Income tax law of india
Income tax law of india
 
I Am A Taxpayer
I Am A TaxpayerI Am A Taxpayer
I Am A Taxpayer
 
direct tax project - income from capital gains detailed study with illustrati...
direct tax project - income from capital gains detailed study with illustrati...direct tax project - income from capital gains detailed study with illustrati...
direct tax project - income from capital gains detailed study with illustrati...
 
Financial planning - effect of cumulative & early saving
Financial planning - effect of cumulative & early savingFinancial planning - effect of cumulative & early saving
Financial planning - effect of cumulative & early saving
 
Indias Agenda for Change - Election 2014
Indias Agenda for Change - Election 2014Indias Agenda for Change - Election 2014
Indias Agenda for Change - Election 2014
 
Lec19
Lec19Lec19
Lec19
 
Lec3
Lec3Lec3
Lec3
 
C:\Fakepath\Tr Epowerpoint
C:\Fakepath\Tr EpowerpointC:\Fakepath\Tr Epowerpoint
C:\Fakepath\Tr Epowerpoint
 
C:\Fakepath\Tr Epowerpoint
C:\Fakepath\Tr EpowerpointC:\Fakepath\Tr Epowerpoint
C:\Fakepath\Tr Epowerpoint
 
Retire rich retire young
Retire rich retire youngRetire rich retire young
Retire rich retire young
 
FISCAL POLICY ECONOMICS FOR GRADE 9 STUDENTS
FISCAL POLICY ECONOMICS FOR GRADE 9 STUDENTSFISCAL POLICY ECONOMICS FOR GRADE 9 STUDENTS
FISCAL POLICY ECONOMICS FOR GRADE 9 STUDENTS
 
Essay On Fundamental Duties Of 250 Words In Hindi
Essay On Fundamental Duties Of 250 Words In HindiEssay On Fundamental Duties Of 250 Words In Hindi
Essay On Fundamental Duties Of 250 Words In Hindi
 
HUMAN TREE CONSULTANCY
HUMAN TREE CONSULTANCYHUMAN TREE CONSULTANCY
HUMAN TREE CONSULTANCY
 
Investing Basics
Investing BasicsInvesting Basics
Investing Basics
 
Gift tax rules 2019 2020 india
Gift tax rules 2019 2020 indiaGift tax rules 2019 2020 india
Gift tax rules 2019 2020 india
 
Its about us and no more about individuals
Its about us and no more about individualsIts about us and no more about individuals
Its about us and no more about individuals
 
Concrete and Whole-Picture Type Indices to Measure Policy Preference over Inc...
Concrete and Whole-Picture Type Indices to Measure Policy Preference over Inc...Concrete and Whole-Picture Type Indices to Measure Policy Preference over Inc...
Concrete and Whole-Picture Type Indices to Measure Policy Preference over Inc...
 

Recently uploaded

SPLICE Working Group: Reusable Code Examples
SPLICE Working Group:Reusable Code ExamplesSPLICE Working Group:Reusable Code Examples
SPLICE Working Group: Reusable Code Examples
Peter Brusilovsky
 
Spellings Wk 4 and Wk 5 for Grade 4 at CAPS
Spellings Wk 4 and Wk 5 for Grade 4 at CAPSSpellings Wk 4 and Wk 5 for Grade 4 at CAPS
Spellings Wk 4 and Wk 5 for Grade 4 at CAPS
AnaAcapella
 
Personalisation of Education by AI and Big Data - Lourdes Guàrdia
Personalisation of Education by AI and Big Data - Lourdes GuàrdiaPersonalisation of Education by AI and Big Data - Lourdes Guàrdia
Personalisation of Education by AI and Big Data - Lourdes Guàrdia
EADTU
 
Transparency, Recognition and the role of eSealing - Ildiko Mazar and Koen No...
Transparency, Recognition and the role of eSealing - Ildiko Mazar and Koen No...Transparency, Recognition and the role of eSealing - Ildiko Mazar and Koen No...
Transparency, Recognition and the role of eSealing - Ildiko Mazar and Koen No...
EADTU
 

Recently uploaded (20)

An overview of the various scriptures in Hinduism
An overview of the various scriptures in HinduismAn overview of the various scriptures in Hinduism
An overview of the various scriptures in Hinduism
 
SPLICE Working Group: Reusable Code Examples
SPLICE Working Group:Reusable Code ExamplesSPLICE Working Group:Reusable Code Examples
SPLICE Working Group: Reusable Code Examples
 
Sternal Fractures & Dislocations - EMGuidewire Radiology Reading Room
Sternal Fractures & Dislocations - EMGuidewire Radiology Reading RoomSternal Fractures & Dislocations - EMGuidewire Radiology Reading Room
Sternal Fractures & Dislocations - EMGuidewire Radiology Reading Room
 
Spellings Wk 4 and Wk 5 for Grade 4 at CAPS
Spellings Wk 4 and Wk 5 for Grade 4 at CAPSSpellings Wk 4 and Wk 5 for Grade 4 at CAPS
Spellings Wk 4 and Wk 5 for Grade 4 at CAPS
 
When Quality Assurance Meets Innovation in Higher Education - Report launch w...
When Quality Assurance Meets Innovation in Higher Education - Report launch w...When Quality Assurance Meets Innovation in Higher Education - Report launch w...
When Quality Assurance Meets Innovation in Higher Education - Report launch w...
 
Mattingly "AI & Prompt Design: Named Entity Recognition"
Mattingly "AI & Prompt Design: Named Entity Recognition"Mattingly "AI & Prompt Design: Named Entity Recognition"
Mattingly "AI & Prompt Design: Named Entity Recognition"
 
How To Create Editable Tree View in Odoo 17
How To Create Editable Tree View in Odoo 17How To Create Editable Tree View in Odoo 17
How To Create Editable Tree View in Odoo 17
 
Graduate Outcomes Presentation Slides - English (v3).pptx
Graduate Outcomes Presentation Slides - English (v3).pptxGraduate Outcomes Presentation Slides - English (v3).pptx
Graduate Outcomes Presentation Slides - English (v3).pptx
 
Personalisation of Education by AI and Big Data - Lourdes Guàrdia
Personalisation of Education by AI and Big Data - Lourdes GuàrdiaPersonalisation of Education by AI and Big Data - Lourdes Guàrdia
Personalisation of Education by AI and Big Data - Lourdes Guàrdia
 
ĐỀ THAM KHẢO KÌ THI TUYỂN SINH VÀO LỚP 10 MÔN TIẾNG ANH FORM 50 CÂU TRẮC NGHI...
ĐỀ THAM KHẢO KÌ THI TUYỂN SINH VÀO LỚP 10 MÔN TIẾNG ANH FORM 50 CÂU TRẮC NGHI...ĐỀ THAM KHẢO KÌ THI TUYỂN SINH VÀO LỚP 10 MÔN TIẾNG ANH FORM 50 CÂU TRẮC NGHI...
ĐỀ THAM KHẢO KÌ THI TUYỂN SINH VÀO LỚP 10 MÔN TIẾNG ANH FORM 50 CÂU TRẮC NGHI...
 
TỔNG HỢP HƠN 100 ĐỀ THI THỬ TỐT NGHIỆP THPT TOÁN 2024 - TỪ CÁC TRƯỜNG, TRƯỜNG...
TỔNG HỢP HƠN 100 ĐỀ THI THỬ TỐT NGHIỆP THPT TOÁN 2024 - TỪ CÁC TRƯỜNG, TRƯỜNG...TỔNG HỢP HƠN 100 ĐỀ THI THỬ TỐT NGHIỆP THPT TOÁN 2024 - TỪ CÁC TRƯỜNG, TRƯỜNG...
TỔNG HỢP HƠN 100 ĐỀ THI THỬ TỐT NGHIỆP THPT TOÁN 2024 - TỪ CÁC TRƯỜNG, TRƯỜNG...
 
male presentation...pdf.................
male presentation...pdf.................male presentation...pdf.................
male presentation...pdf.................
 
Spring gala 2024 photo slideshow - Celebrating School-Community Partnerships
Spring gala 2024 photo slideshow - Celebrating School-Community PartnershipsSpring gala 2024 photo slideshow - Celebrating School-Community Partnerships
Spring gala 2024 photo slideshow - Celebrating School-Community Partnerships
 
OSCM Unit 2_Operations Processes & Systems
OSCM Unit 2_Operations Processes & SystemsOSCM Unit 2_Operations Processes & Systems
OSCM Unit 2_Operations Processes & Systems
 
Đề tieng anh thpt 2024 danh cho cac ban hoc sinh
Đề tieng anh thpt 2024 danh cho cac ban hoc sinhĐề tieng anh thpt 2024 danh cho cac ban hoc sinh
Đề tieng anh thpt 2024 danh cho cac ban hoc sinh
 
Transparency, Recognition and the role of eSealing - Ildiko Mazar and Koen No...
Transparency, Recognition and the role of eSealing - Ildiko Mazar and Koen No...Transparency, Recognition and the role of eSealing - Ildiko Mazar and Koen No...
Transparency, Recognition and the role of eSealing - Ildiko Mazar and Koen No...
 
Andreas Schleicher presents at the launch of What does child empowerment mean...
Andreas Schleicher presents at the launch of What does child empowerment mean...Andreas Schleicher presents at the launch of What does child empowerment mean...
Andreas Schleicher presents at the launch of What does child empowerment mean...
 
ANTI PARKISON DRUGS.pptx
ANTI         PARKISON          DRUGS.pptxANTI         PARKISON          DRUGS.pptx
ANTI PARKISON DRUGS.pptx
 
Mattingly "AI and Prompt Design: LLMs with NER"
Mattingly "AI and Prompt Design: LLMs with NER"Mattingly "AI and Prompt Design: LLMs with NER"
Mattingly "AI and Prompt Design: LLMs with NER"
 
How to Manage Website in Odoo 17 Studio App.pptx
How to Manage Website in Odoo 17 Studio App.pptxHow to Manage Website in Odoo 17 Studio App.pptx
How to Manage Website in Odoo 17 Studio App.pptx
 

English

  • 1. OMPREHENSION Q.1-10. Read the following passage carefully and answer the questions given below it. You’ve got a sarkari naukri, a government job, whether you know it or not. Or whether you like it or not. Sarkari naukri doesn’t mean that you have to be a government babu, or an MLA, or MP, or a member of the judiciary or of the defence forces, or an employee of a PSU. You can be a teacher, or an accountant, or a marketing executive, or an event manager, or a homemaker, or a student, but you are still a full-time government employee. By sarkari naukri I’m not talking about the much-touted and equally criticised NREGS. The sarkari service that compulsorily we all do, whether we know it or not, is - unlike NREGS - purely honorary, in the sense that we don’t get paid for it. On the contrary, all of us unwitting government servants, in one way or the other, pay the government for the privilege of our having been given an unpaid government job to do. If you belong to the 2% of the population of this country which pays income tax you are an unpaid sarkari servant. For if you have a taxable income you are not only obliged to pay a tax on it but you are also obliged to file returns to prove that you have indeed paid the tax due from you. In other words, you not only pay your tax but you also become an unpaid tax assessor-cum-collector for the government because the onus of proof is on you to show that not only have you paid your tax but that you have done the government’s work of having updated the government’s records which show you’ve paid the tax. Unless you are an accountant yourself, you have to pay an accountant to file your tax returns for you. (Have you ever tried to figure out a tax return form? Instead of SAT and GMAT and all those other tests to gauge students’ mental abilities they should ask candidates to fill in an average sarkari tax form. Any non-specialist who can do this has a genius-level IQ and is a potential member of Mensa.) So you’ve not only paid your tax, and also done the government’s unpaid work of filing your returns, but on top of that you’ve shelled out an accountant’s fee for having done the government’s job for it. You’ve paid for the privilege of having sarkari naukri. OK, so you don’t have a taxable income. No matter. You’re still eligible for sarkari naukri. Each time you drink a cup of tea or coffee, travel by bus or car, go for a medical check-up, buy anything from chalk to cheese, from an apple to a zither and anything in between you’re raising revenues for the government through a spiderweb of taxes including sales taxes, excise duties, octroi, et taxcetera, et taxcetera. Which means that you are an unofficial employee of the sarkar (why else would you be raising funds for it?), which means you are a sarkari naukar-chaakar. The sarkar taxes almost everything of daily use, for all of us, from the richest to the poorest. What was that you said? The only thing left untaxed is love and fresh air? Uh, oh. You might have just given the FM a couple of taxation ideas for his next budget. Our sarkar’s motto is: Ask not what your sarkar can do for you; ask what you can do for your sarkar. As more and more of us become the unpaid slaves of the sarkar, the sarkar itself does even less and less. Which means that we, the sarkari karamcharis, have to do more and more. And our sarkari naukri is for life. There’s no such thing as retirement. And that’s a sarkari guarantee, like it or lump it. Lump is more like it. Q.1. What is Sarkar’s motto ? (1) To ensure a good living for every citizen (2) To ask what a citizen can do for it, not what it can do for the citizens (3) Levy taxes on the commodities (4) Raise the standard of living (5) None of these Q.2. How does the author say that one can be an “unpaid Sarkari Servant” ? (1) By paying income tax regularly (2) If we are among 2% of the Indian population. (3) By shelving the accountant’s fee for filing returns. (4) All 1,2 and 3 (5) Only 1 and 2 Q.3. How does the author say that even if you do not have a taxable income you are a “Sarkari Servant” ? (1) By traveling by bus or car (2) By raising revenues
  • 2. (3) By paying indirect taxes on the commodities we use or buy. (4) All 1,2 and 3 (5) None of these Q.4. How according to the author is sarkari naukri for life ? (1) Because there is a sarkari guarantee for us. (2) As there is no retirement in this job. (3) As the job offers us life time service (4) Because the government will continue taking taxes from us (5) None of these Q.5. How is paying tax also becoming an unpaid tax assessor-cum-collector for the government ? (1) By being a part of the NREGS (2) As by paying tax we are indirectly updating the government’s records. (3) By being an accountant yourself (4) By filing your returns yourself (5) All of the above Q.6. Which of the following are the taxes mentioned in the passage ? (1) Sales taxes, excise, duties, octroi, et taxcetra , etaxcetra (2) Sales tax, excise duties octroi, et taxcetra (3) Sales tax, excise duties, octroi (4) Sales tax,exicise duties, octroi, extaxcetra (5) Sales tax, excise duties, et taxcetra Q.7. What does the author show by this passage ? (1) Importance of government services (2) Taxes and their role in the country (3) Heavy burden of taxes on all the citizens (4) All 1,2 and 3 (5) Only 2 and 3 Q.8. What does the following statement mean – “which means that, we, the Sarkari Karamcharis, have to do more and more” ? (1) The government servants have to work harder (2) The citizens have to pay more taxes to the government every year. (3) We the citizens of India have to pay taxes regularly (4) All 1,2 and 3 (5) None of these Q.9. Who are the “unpaid slaves” ? (1) People in government services (2) Citizens of the country (3) Each and every citizen of the country (4) Persons working under the NREGS scheme (5) People who are not necessarily in the government service but pay taxes. Q.10. Which of the following is TRUE is context of the passage ? (1) 2% of the population of our country is under the tax free income (2) Indian government levies taxes on all the commodities (3) People who are not officially government servants are free from tax. (4) All the citizens are full time government servants indirectly (5) None of these
  • 3. ANSWERS: Q.1.(2) Q.2.(1) Q.3.(3) Q.4.(4) Q.5.(2) Q.6.(3) Q.7.(2) Q.8.(5) Q.9.(5) Q.10.(5) --------------------------------------------------------------------------------------------------------------------------------------- -------------------------------------------- FILL IN THE BLANKS Q.1-5. In each of the following sentences there are two blank spaces. Below each five pairs of words have been denoted by numbers (1), (2), (3), (4) and (5). Find out which pair of words can be filled up in the blanks in the sentences in the same sequence to make the sentence meaningfully complete. Q.1. Despite their strong resentment, the tribals ———— the new laws as long as the government officials did not ———— them too strenuously. (1) welcomed, observe (2) tolerated, enforce (3) amended, follow (4) laid, punish (5) absorb, effect Q.2. He gave me an additional ——— of Rs. 200 a month by ——— of the commissioner. (1) allowance, permission (2) pay, allowance (3) salary, admittance (4) payment, agreement (5) benefit, laws Q.3. He was too ———— to ————— that he had been wrong. (1) docile, accept (2) polite, agree (3) nervous, rejoice (4) obstinate, admit (5) determined, promise Q.4. I am not —— to sell you my house unless you offer a more ——— price. (1) conform, true (2) prepared, realistic (3) ready, correct (4) great, achieved (5) accepted, standard Q.5. Since we ——— read every book, we ——— only the famous ones. (1) have, sold (2) should, buy (3) must, ignore (4) can, purchase (5) cannot, select ANSWERS Q.1.(2) Tolerated here means beared the laws and enforce means brought into action or force.
  • 4. For other options:- - Welcomed means accepted openly or happily and observe means to study something closely. - amended means changed, it can not be used here as the tribals can not change the laws. - laid means mode or brought into form. The tribals will not lay the laws of the government. Q.2.(1) allowance means a sum of money granted for a particular purpose and the word permission is compatible with it. The other options pay, salary and payment can not be used here with the word additional. Q.3.(4) obstinate means firm or stubborn, generally used in a negative sense and admit means accept. For other options:- - docile means easily managed or handled it is inappropriate here. - determined means dedicated towards something but the word promise is not suitable in the latter part of the sentence. - polite and nervous do not present a negative sense here so they cannot be used. Q.4.(2) Prepared means ready mentally and realistic means having a sensible and practical aspect. Q.5.(5) Cannot means not able to and select means choose, as we cannot read every book so we select only the famous ones. --------------------------------------------------------------------------------------------------------------------------------------- -------------------------------------------- SENTENCE COMPLETION Q.1-5. In each of the following sentences there is a blank space given. Below each statements have been denoted by numbers (1), (2), (3), (4) and (5). Find out which statement can be filled up in the blank in the same sequence to make the sentence meaningfully complete. Q.1. BRICS is a group of emerging economies of the world. The word BRIC was originally coined by JIM O’meiu of Goldman Sachs in 2001. Goldman Sachs is an international financial company which conducts research in the future growth of global economy. ___________ (1) Global economy generally refers to the economy, which is based on economies of all of the worlds countries. (2) Goldman Sachs has identified Brazil, Russia, China and India as the four rising economies. (3) It appears that South Africa has been included in BRICK due to strategic regions. (4) Research and development is and integral part of every organization. (5) Optimism is difficult when considering an unpredictable future. Q.2. It has become the latest fashion not only in political circles but also in print and media circles to blame Armed Forces Special Powers Act for all the unrest turbulence ———— very. Conveniently they all overlook the undeniable fact that Pakistan after suffering a crushing defeat from India in 1971 has disclosed an open proxy war against India. (1) and large scale violence not only in just Kashmir valley but also in many other parts like Manipur. (2) Parliament after serious deliberations want ahead to enact the AFSPA on September 11,1958 to safeguard the armed Forces. (3) AS terrorism raise its ugly head in Jammu and Kashmir in late eighties, Parliament is self and not Army felt compelled to enact the act. (4)The AFSPA was being unnecessarily demonized although it had nothing to do with the present unrest in Kashmir. (5) The Armed Forces are not only deployed to work in hostile environment but also in a difficult terrain. Q.3. You must create a clear vision of your target and never lose sight of it. -————. Your incentive loses appeal, efforts slacken and you make no progress.
  • 5. (1) Psychological laws usually are not so dramatic. (2) The practice of keeping your eyes open is complicated. (3) Once you allow your gaze to wander, good intention fade. (4) They gather details of anybody’s private life for the safe purpose of passing on scandalous tidbits to others. (5) In everyday conversation, we speak of looking forward to some pleasure. Q.4. While preparing a presentation, the beginning should be carefully designed to get attention. A shock, a story, humor, question or facts and figures can achieve this purpose for you. Howewer, you should be so confident and well rehearsed that they look natural —— (1) The key points in the middle section of your presentation should be logical. (2) Ensure that your are audible to all the audience. (3) Keep your comments brief and avoid being caught extempore at only point or question. (4) Make sure that your visual aid is clearly visible and legible to all the audience. (5) By mixing them at intervals in your presentation you will be able to hold their interest and communicate your message successfully. Q.5. ——————. It deals with those aspects even if it is minor or major but essential for building the constitution. If a person has studied it deeply he will be able to understand instance of constitution. (1) What is of supreme importance is that parliamentarians must be totally honest. (2) Politics is very crucial part of nation as it involves each person of society for framing any law, policy or framework. (3) India being a democratic nation has the viability of accepting the right and efficient person as their representative and it must be fulfilled for welfare of people. (4) A member of parliament should be at least post graduate and that too in Human Resources or Marketing. (5) A public representative ought to have sensible mind with high degree of zone of tolerance and attitude of positive progressive thinker. ANSWERS Q.1.(2) The passage is talking about BRICK and how the concept come up along with stating what Goldman sachs is. Q.2.(1) The writer is talking about AFSPA and how it is perceived by the society at large. Q.3.(3) The author is trying to tell us how we should have a clear picture of our target and never loose over determination. Q.4.(5) Here it is being told how one should prepare and deliver a presentation and the various ways to make it effective. Q.5.(2) The writer speaks on Politics, its pros and cons and what a person can gain by learning it. --------------------------------------------------------------------------------------------------------------------------------------- -------------------------------------------- SENTENCE IMPROVEMENT Q.1-5. Which of the Phrases (1), (2) (3) and (4) given below each sentence should replace the phrases printed in bold in the sentence to make it grammatically correct ? If the sentence is correct as it is given and no correction is required, mark (5) as the answer. Q.1. Of the various sectors of the Indian economy, the agriculture sector is the one that have large witnessing the era of modernization. (1) have large witnessed (2) was large witnessing (3) were largely witnessed (4) has
  • 6. largely witnessed (5) No correction required Q.2. The primary objective of the association is being overthrow the British Raj in India and establish a national republic passed on freedom and equality. (1) was being (2) to be (3) was to (4) were to be (5) No correction required Q.3. The past two decades see several attemptings to set up economic alliances, but they all failed. (1) see several attempt (2) saw several attempts (3) view several attempted (4) see several attemps (5) No correction required Q.4. The report suggested that it was only those at the bottom of the economic ladder who suffered through the recent recession years. (1) who were suffered (2) who was suffered (3) who is suffered (4) who for suffered (5) No correction required Q.5. During the crises, the prompt actions of policymakers across the globe was for being critical in averting a much deeper disaster. (1) was critical (2) were critical (3) to critical (4) being (5) No correction required ANSWERS: Q.1.(4) Q.2.(3) Q.3.(2) Q.4.(5) Q.5.(2) --------------------------------------------------------------------------------------------------------------------------------------- -------------------------------------------- ERROR DETECTION Q.1-5. Read each sentence given below and find out whether there is an error in it. The error if any will be one of the parts of the sentence which are marked as 1,2,3 and 4. If there is no error, the answer will be (5) i.e. No error. (Ignore the errors of punctuation, if any) Q.1. After finishing(1)/ his speech(2)/ the audience was invited(3)/to ask questions(4)/. No error(5) Q.2. The little child(1)/was sitting closely(2)/beside(3)/his teacher(4)/. No error(5) Q.3. Built on 230 acres(1)/the palace of Udaipur(2)/is one of the (3)/showplaces of Rajasthan(4)/. No error(5) Q.4. Tonight(1)/ I am going to check(2)/that Sanjeev(3)/will do his homework correctly(4)/. No error(5) Q.5. Due to the house(1)/ sparkled with flames(2)/the fireman used the water(3)/to extinguish the fire(4)/. No error(5) ANSWERS Q.1.(5) No error. Q.2.(2) Use ‘close’ in place of closely here Q.3.(5) No error Q.4.(4) This sentence talks about the person ensuring that Sanjeev does his work correctly For personal plans or predictions we use “going to” with the simple present tense.
  • 7. Q.5.(1) Use “as” in place of due to, “Due to” is followed by a noun. -------------------------------------------------------------------------------------------------------------------------------------- --------------------------------------------- SENTENCE EVALUATION Q.1-4. In each of these questions there are three sentences given as(I),(II),(III),find out which two or three sentences convey the same meaning.If all three sentences are different, mark (5) as the answer i.e ‘all different’. Q.1. I. The Manager would like you to help him in locating the fault. II. If you help him in locating the fault , the Manager would like you. III. The Manager desires that you should provide him the necessary assistance to locate the fault. (1) I and II (2) II and III (3) I and III (4) All are same (5) All are different Q.2. I. Although strike of transporters continues, I shall come. II. I shall come if the strike of transporters continues. III. Even though I come, the strike of transporter is going to continue. (1) I and II (2) II and III (3) I and III (4) All are same (5) All are different Q.3. I. Unlike the tribals who are hard working the urban communities cannot withstand physical strain. II. The tribals can withstand physical strain where as urban communities cannot. III. Because the tribals are hardworking they can tolerate physical strain. (1) I and II (2) II and III (3) I and III (4) All are same (5) All are different Q4. I. If you need a visa ,you must submit an application along with your passport and a copy of income tax returns. II. Unless you submit an application along with your passport and copy of income tax returns you will not get visa. III. If you submit your application along with your passport and a copy of income tax returns you do not need visa. (1) I and II (2) II and III (3) I and III (4) All are same (5) All are different ANSWERS Q.1.(3) Both (1) and(3) convey the same meaning Q.2.(5) All the three statements convey different messages. Q.3.(5) All the three statement have different meaning so (5)is the answer. Q.4.(1) Both (1)and (2) sentences give the same meaning that for getting visa, passport and copy of income tax returns are mandatory.
  • 8. --------------------------------------------------------------------------------------------------------------------------------------- -------------------------------------------- ERROR DETECTION Q.1-5. Read each sentence to find out if there is any error in it. The error, if any, will be in one part of the sentence. The number of that part is the answer. If there is no error, the answer is (5). Q.1. If you will buy one box(1)/ at the regular price(2)/ you will receive another(3)/ at no extra cost (4)/ No Error (5) Q.2. The train(1)/ has arrived (2)/ when he reached(3)/ the station (4) /No Error (5) Q.3. Daniel was(1)/one of the greatest(2)/ judges that(3)/has ever lived(4)/ No Error (5) Q.4. The climate of(1)/ Goa is (2)/more pleasant(3)/ than Inland resort(4) / No Error (5) Q.5. She presented (1)/ him(2)/two dozens (3)/pens(4)/ No Error (5) ANSWERS Q.1.(1) "Will" should replace omitted because if is not followed by "will". Q.2.(2) "Had " should replace"has" because the sentence is in past tense. Q.3.(4) "Have" should replace "has" because one of than Noun (plural) who/which/ that+verb(plural). Q.4.(4) "Than" should replace followed by "than of ". Q.5.(3) --------------------------------------------------------------------------------------------------------------------------------------- -------------------------------------------- FILL IN THE BLANKS Q.1-5. Fill in the blanks with appropriate words in both the blanks Q.1. The petitioner had ____ an immediate stay from the court on allotment of the flats till ____ of investigation into the alleged irregularities. (1) asked, process (2) sought, completion (3) propagated, finish (4) demanded, course (5) granted, end Q.2. Modern ‘music, doesn’t lend ______ to longevity; Sometimes it is hand to remember what the biggest hit from a _____ of years ago was. (1) itself,couple (2) must, few (3) money, some (4) matter, spatter (5) substance, vacant Q.3. Opportunities_____ when they are seized and _____when they are not . (1) gather, evade (2) needed, reflect (3) create, disperse (4) shout, calm (5) multiply, die Q.4. At the sound of bell we are ____to pause, relax our body and become gently _____of our in- breath and out-breath. (1) asked, thoughtless (2) requested, wishful (3) invited, aware (4) forget, meaningful
  • 9. (5) braced,calm Q.5. ______to listen to the sounds of nature, we find that we can afterwards carry on with whatever we were doing in a more _____ and loving way. (1) feeling, helpful (2) waiting, rough (3) dreaming, dramatic (4) pausing, attentive (5) hurting, creative ANSWERS Q.1.(2) Q.2.(1) Q.3.(5) Q.4.(3) Q.5.(1) --------------------------------------------------------------------------------------------------------------------------------------- -------------------------------------------- REARRANGEMENT Q.1-5.Rearrange the following sentences (A),(B),(C), (D), (E) and (F) in the proper sequence to form a meaningful paragraph, then answer the questions given below them. (A) The students were introduced to make a working model of cars using solar energy (B) The students were elated after successfully completing the working model of the car (C) But in a matter of one hour, their nimble hands transformed (D) At a time when the significance of research activity shifted to the field of clean energy (E) These parts into small and beautiful cars that were powered by solar energy. (F) They were given wires, plastic parts, tiny wheels and solar panels Q.1. Which of the following would be the SECOND statement after rearrangement? (1) C (2) A (3) D (4) F (5) E Q.2. Which of the following would be the SIXTH statement after rearrangement? (1) A (2) F (3) D (4) B (5) C Q.3. Which of the following would be the THIRD statement after rearrangement? (1) C (2) A (3) D (4) F (5) E Q.4. Which of the following would be the FIRST statement after rearrangement? (1) C (2) F (3) E (4) B (5) D Q.5. Which of the following would be the FIFTH statement after rearrangement? (1) E (2) A (3) D (4) B (5) C ANSWER Q.1.(2) Q.2.(4)
  • 10. Q.3.(4) Q.4.(5) Q.5.(1) The arrangement is DAFCEB The paragraph is about an assignment which was given to students for making cars using solar energy which brought them contentment and happiness --------------------------------------------------------------------------------------------------------------------------------------- -------------------------------------------- FILL IN THE BLANKS Q.1-5.In each of the following sentences there are two blank spaces. Below each five pairs of words have been denoted by numbers (1), (2), (3), (4) and (5). Find out which pair of words can be filled up in the blanks in the sentences in the same sequence to make the sentence meaningfully complete. Q.1. Infrastructure services -----------to poverty reduction and improvements in living -------in several ways. (1) confer, segment (2) contribute, standards (3) subsidize, style (4) extend , quality (5) regulate, organisms Q.2. The organic mangoes are------------- without using any chemical manure, pesticide, but are -------- ---naturally. (1) flourished, seasoned (2) evolved, flowered (3) grown-up, developed (4) matured, bloomed (5) grown, ripened Q.3. Hafiz ----------- faces and sells flags branded with the IPL teams to -----------a meal for his family (1) paints, earn (2) makes, manages (3) coloured, obtain (4) drew, secure (5) read, receive Q.4. Groups of auto lifters were -----------and found to have changed the colour of ---------bikes. (1) spotted, repair (2) exposed, borrowed (3) seized, bought (4) caught, stolen (5) grabbed, maintained Q.5. People --------difficulties in reaching their destinations on ---------- due to traffic jam (1) suffered, travel (2) challenged, arrival (3) faced, time (4) experienced, demand (5) sustained , crow ANSWERS Q.1.(2) Contribute, standards For other options: Confer (v)- give Subsidize (v)-contribute Regulate (v)- control Q.2.(5) Grown, ripened For other options: Seasoned (v)- made tasty Evolved (v)-emerged Q.3.(1) Paints, earn Q.4.(4) Caught, stolen
  • 11. Q.5.(3) Faced, time --------------------------------------------------------------------------------------------------------------------------------------- -------------------------------------------- ERROR DETECTION Q.1-5. Read each sentence to find out if there is any error in it. The error, if any, will be in one part of the sentence. The number of that part is the answer. If there is no error, the answer is (5). Q.1. Police are keeping an eye on the top notch bookies (1)/ as they would get evenly more active (2)/ with two matches of IPL (3)/ taking place in the city (4)/ No error (5) Q.2. The saplings, kept in nurseries (1)/ are ageing and the roots (2)/ are begin to grow out of plastic containers (3)/ and getting into the ground (4)/ No error (5) Q.3. Fire personnel tried its best (1)/ to prevent spread of the fire (2)/ in the congested locality (3)/adjacent to the affected godown (4)/ No error (5) Q.4. It is in the interest of the customers to provide (1)/ their contact address and telephone numbers (2)/ so that the banks can pass on any information (3)/ about their services and products (4)/ No error(5) Q.5. The bamboo umbrellas, which used to shelter pundits and sages (1)/ on the banks of Ganga (2)/ are being replaced artificial umbrellas (3)/ made up of plastic (4)/ No error (5) ANSWERS Q.1.(2) ‘Even’ should replace ‘evenly’- an adverb only would qualify another adverb ‘more’ Q.2.(3) ‘Beginning’ should replace ‘begin’ – present continuous tense is to be used Q.3.(1) ‘Their’ should replace ‘its’ as possessive pronouns according to ‘fire personnel’ Q.4.(5) Q.5.(3) ‘Replaced with’ should be used- ‘replaced’ is always followed by ‘with’ --------------------------------------------------------------------------------------------------------------------------------------- -------------------------------------------- MISAPPROPRIATION Q.1-5.In each question below a sentence with four words printed in bold type is given. These are numbered (1),(2),(3) and (4) . One of these four words printed in bold may be either wrongly spelt or inappropriate in context of the sentence. Find out the word which is wrongly spelt or inappropriate if any. The number of that word is your answer. If all the words printed in bold are correctly spelt & also appropriate in the context of the sentence, mark (5) "All correct" as your answer. Q.1. Without any prior notice to the traders, a portion of the road has been barricaded to (1) (2) (3) undertaken repair work All correct (4) (5) Q.2. The elephant had fallen sick the day after being tranquilized by foresters and veterinary (1) (2) (3) doctors were called in to treat it All correct
  • 12. (4) (5) Q.3. Young girls as well the grown-up ones are now looking to learn various techniques to keep (1) (2) (3) themselves protected All correct (4) (5) Q.4. Some of the people running small sideroad businesses, like taking children for horse rides, (1) (2) (3) are misbehaving with kids All correct (4) (5) Q.5. It requires highest measures of creativity to manufacture a play which engages the attention (1) (2) (3) (4) and thoughts of the viewers All correct (5) ANSWERS Q.1.(4) ‘Undertake’ should be used – after ‘to’ first form of verb is to be used Q.2.(5) Q.3.(1) ‘As well as’ should be used which means ‘also’ Q.4.(2) ‘Roadside’ is the correct word Q.5.(2) ‘Produce’ should be used – which would mean ‘present a play’ --------------------------------------------------------------------------------------------------------------------------------------- -------------------------------------------- READING COMPREHENSION Q.1-10. Read the following passage carefully and answer the questions given below it. The Aadhaar scheme of the Unique Identification Authority of India (UIDAI) is to provide India’s billion- plus people with a unique identification number. Enrolment is not mandatory, though it was mentioned that it would be difficult for people to access public services if not done. The scheme requires individuals to provide their photograph, fingerprints and iris scan along with documentary personal information for data capture by outsourced operators. It is meant to bypass the corrupt bureaucratic system and deliver government subsidies and grants to the poor, and bring them into the banking system. Sceptics argue that it is an effort to capture the funds of hundreds of millions of micro- and nano-investors who are today outside the banking system, to bring them into the credit economy. The scheme was introduced as a pilot project in Karnataka’s Mysore district. The poor and those who survive on daily wages were not enthusiastic about enrolment, because it meant losing four or five days wages, to stand in queues, to fill up forms, to produce documents, to provide biometrics, etc., and, later, to open bank accounts. The UIDAI overcame the initial reluctance by wide advertisement of the benefits of enrolment. When this too did not achieve the target set, the local administration informed the public that PDS ration and LPG supply would not be available without the Aadhaar number. This resulted in serpentine queues right through the day at enrolment centres, at the end of which the UIDAI could claim that 95 per cent of Mysore district’s population had enrolled itself into the scheme. Media reports indicate that commencing January 1, 2013, MGNREGA, the Rajiv Gandhi Awas Yojana
  • 13. (RGAY), the Ashraya housing scheme, Bhagyalakshmi and the social security and pension scheme will be linked with Aadhaar in Mysore district. This linking, with rights like salary and pension, and important entitled benefits and services, has raised some hackles because enrolment is not mandatory. It has led to questions on whether salary and pension rights, and benefits like PDS ration and LPG supply can be denied just because an individual does not possess a unique Aadhaar number. Today, teachers in Maharashtra and government employees in Jharkhand cannot draw their salaries. Apart from pro-poor projects like MGNREGA and RGAY, even jobs, housing, provident funds and registering a marriage now require enrolment. From being not mandatory, the “poor-inclusive” .Aadhaar scheme appears to have quietly metamorphosed into becoming exclusionary and non- optional. The UIDAI’s own Biometrics Standards Committee stated that retaining biometric efficiency for a database of more than one billion people “has not been adequately analysed” and the problem of fingerprint quality in India “has not been studied in depth.” Thus the technological basis of the project remains doubtful. Q.1. Which of the following are required for enrolment for ‘Aadhar scheme’? A. Personal information B. Iris scan C. Photograph D. Fingerprints (1) Only A and C (2) Only B and D (3) Only B and C (4) All except A (5) All of the above Q.2. Why, according to the author, are the poor not interested in ‘Aadhar scheme’? (1) They are not willing to provide their biometric measures (2) They do not possess the required documents which are needed to be produced at the time of enrolment (3) They do not perceive it as a benefit to them (4) It involves a number of procedures which are to be completed and they will have to sacrifice their daily work schedule (5) The poor are interested in enrolling into ‘Aadhar scheme’ Q.3. Which of the following questions has been raised by the author? (1) Why did ‘Aadhar scheme’ not prove to be useful? (2) Can a person be denied benefits like PDS ration and LPG supply because he does not have Aadhar card? (3) Why is Aadhar enrolment not mandatory? (4) Why can’t government employees in Jharkhand draw their salary? (5) No question has been raised by the author Q.4. What is meant by ‘Biometric efficiency’ as mentioned in the passage? (1) Correct biometric measurement (2) Biometric analysis (3) Personal efficiency (4) Biometric system (5) Biological specifications Q.5. Which scheme is NOT linked with Aadhar scheme in Mysore district? (1) Rajiv Gandhi Swarna Jayanti swarojgar yojna (2) Ashraya housing scheme (3) Social security and pension scheme (4) MGNREGA
  • 14. (5) All of the above Q.6. ———————— does not require Aadhar enrolment. (1) Housing (2) Admission in college (3) Registering a marriage (4) Provident funds (5) Jobs Q.7. What was declared initially about ‘Aadhar scheme’? (1) One can get enrolled into ‘Aadhar’ online and Aadhar card would be uploaded on the website (2) One can draw pension or salary only when one has Aadhar card (3) Aadhar card would serve to prove one’s identity and is nontransferable (4) One would not get access to public services if he does not get enrolled into ‘Aadhar scheme’ (5) Not mentioned in the passage Q.8. Which of the following is FALSE according to the passage? (1) Fingerprint quality in India has not been studied properly (2) Aadhar scheme was introduced to provide with a universal identification number (3) Technological basis of the project cannot be established (4) The scheme was introduced as a pilot project in Karnataka’s Mysore district (5) All of the above Q.9. What led UIDAI to claim that 95 per cent of Mysore district’s population had enrolled itself into the scheme? (1) When teachers and government employees could not draw their salaries (2) When benefits of Aadhar were publicized widely (3) When it was declared that PDS ration and LPG supply would be available to only Aadhar card holders (4) When the queues at enrolment centres became longer (5) All of the above ANSWERS: Q.1.(5) Q.2.(4) Q.3.(2) Q.4.(1) Q.5.(1) Q.6.(2) Q.7.(4) Q.8.(2) Q.9.(3) --------------------------------------------------------------------------------------------------------------------------------------- -------------------------------------------- CLOZE TEST
  • 15. Q.1-10. In the following passage there are blanks each of which has been numbered. These numbers are printed below the passage and against each five words have been suggested, one of which fits the blanks appropriately. Find out the appropriate word in each case. A student looking for personal attention ends up in a tutoring class that is no less crowded. With no regulation, the (1) could go up to a hundred. Many coaching institutions hesitate to include “average” students. Like a shadow, private tuitions go on in tandem from KG to overseas degrees. Private coaching institutions (2) the prevailing demand for education. Their success comes from the government’s “inability to provide education to everyone who desires it, provide education of quality perceived to be (3).” These centres promise students the limited seats in credible higher institutions and social support for them is pledged by parents (4) to meet the high cost Many parents are unable to help children with home work at specialised levels. Added to these, are the lengthy curriculum, scarcity of efficient teachers, communication gap between teachers and students, disturbed academic schedules, co-curricular activities that takes the students away from classrooms, tuitions that are seen as a status symbol, (5) of some subjects over others, faulty school administration in allotting classes and faulty government policies. Private coaching seems the only (6) Teaching is bad in general and, coaching ensures (7) the exams. Private tutors keep students busy during off hours and, (8) unruly ones. If such centres are bad, why haven’t the authorities taken (9) steps? Compared to a lot of schools, aren’t coaching institutes better in terms of teaching, dedication of instructors, student- teacher relationship and norms of conducting mock tests? In the current academic environment, private tuitions are a practical (10). Though there seems to be varied reasons for students to seek private tutoring, the predominant one is the need (or craze) to score better. Q.1. (1) strength (2) reach (3) crowd (4) intelligence (5) expertise Q.2. (1) maneuver (2) impose (3) ditch (4) exploit (5) debilitate Q.3. (1) economical (2) pleasing (3) questionable (4) detrimental (5) worthwhile Q.4. (1) incline (2) willing (3) leave (4) opt (5) hesitant Q.5. (1) partially (2) improvement (3) preference (4) alternative (5) blessing Q.6. (1) solution (2) way (3) clarification (4) feedback (5) complication Q.7. (1) preparing (2) passing (3) through (4) developing (5) excelling Q.8. (1) cherish (2) submissive (3) temper (4) terminate (5) tame Q.9. (1) valid (2) agile (3) effective (4) restless (5) haste Q.10. (1) essence (2) excuses (3) constraint (4) necessity (5) obsessed ANSWERS Q.1.(1) Strength For other options: Expertise (n)- skill Q.2.(4) Exploit For other options: Maneuver (v)-plan Impose (v)-dictate Debilitate (v)-exhaust Ditch (v)-elude Q.3.(5) Worthwhile For other options: Economical (adj)-frugal
  • 16. Detrimental (adj)-damaging Q.4.(2) Willing For other options: Opt (v)- choose Hesitant (adj)-reluctant Q.5.(3) Preference For other options: Partially (adv)- not completely Alternative (n)- choice Q.6.(1) Solution For other options: Clarification (n)-description Feedback (n)-response Q.7.(2) Passing Q.8.(5) Tame For other options: Submissive (adj)-compliant Cherish (v)- enjoy Temper (n)-attitude Q.9.(3) Effective For other options: Agile (adj)- quick Restless (adj)- untiring Valid (adj)- logical Q.10.(4) Necessity For other options: Essence (n)-significance Obsessed (adj)-captivated --------------------------------------------------------------------------------------------------------------------------------------- -------------------------------------------- SENTENCE COMPLETION Q.1-4.In each of the following questions a short passage is given with one of the lines in the passage missing and represented by a blank. Select the best out of the five answer choices given to make the passage complete and coherent. Q.1. Sipping endlessly from that hot mug of tea or coffee might soon be a thing of the past with climate change. Tea is dependent on the distribution of rainfall and its pattern. The impact of global warming is immense and there is a steady decline in productivity,—————————. Tea production is mostly helped by land cover, but climatic factors have been playing spoilsport for a while. Secondary variables such as pests and diseases and management policies too play deterrents. (1) It has not affected coffee production so far as the crop is largely grown under the shade of trees (2) On an average, 108 million kg tea is contributed by South Indian tea gardens, as per records (3) Elsewhere in the world, climate change is likely to drastically affect the production of coffee (4) However, all tea-growing regions across the world are experiencing adverse climatic and weather conditions (5) Where agricultural practices depend on rain-fed conditions Q.2. Police have started filing FIRs against overloaded heavy vehicles —————————————
  • 17. ——. Police have launched this initiative to control accidents caused by heavy vehicles that are found responsible in most road accident cases. Overloaded vehicles lead to destruction of road surfaces which in turn cause traffic jams and accidents. (1) Those heavy vehicles were rashly driven and overloaded beyond their capacity (2) Driving through expressway during night is fraught with danger due to the heavy vehicles (3) Under the Prevention of Damage to Public Property (PDPP) Act and several sections of the India Penal code (IPC). (4) The first week of April, cops have been filing FIRs against three-four dumpers on a daily basis. (5)They indulge in speeding and frequent lane changes to reach their destinations. Q.3. Bharathi Lions Talking Library is a treasure trove not just for visually-challenged school and college students but also for researchers and professionals. More than 50 visually-challenged people visit the ‘talking library’ every week to get the recorded cassettes or get their notes recorded. ———— ——————————————————-The library, which opened five years ago, has around 1,300 cassettes on school and college subjects. (1) Books recorded at the talking library so that he could listen to them later (2) Without learning Braille, the students will not be able to understand spellings and the language which will be crucial in their career later (3) We either recite the contents or depute volunteers for the task (4) There is a librarian who records it for them or gets someone else to record it for them. (5) With advent of many technological tools to help the visually-challenged students, experts say, learning of Braille is slowly becoming a passé Q.4. —————————————————————————————, child counsellors suggest them to mend the same in order to keep a check on their wards behavioral development and outlook towards life in general. With the younger generation spending a good amount of their time on virtual space, parents have much to catch up on their kid’s life. Until and unless that gap between the kid and the parents isn’t bridged, wherein parents are supposed to talk about anything and everything with their children, kids will never ever confide themselves (1) While teenage girls are easy to communicate with, boys are usually a hard nut to crack. (2) Citing communication gap as a major reason of divide between children and their parents (3) They should not impose opposing views directly on to the kids (4) The idea is to make the kid talk first and then take them on the path of persuasion (5) We need to be their friends and then engage in an open communication with them ANSWERS: Q.1.(5) The paragraph is about the effect of global warming on tea production Q.2.(3) The paragraph is about the new initiative of police to check overloaded heavy vehicles Q.3.(4) The paragraph describes the services provided by Bharathi Lions Talking Library for visually- challenged people Q.4.(2) The paragraph described child counselors suggestion to bridge the gap between children and their parents --------------------------------------------------------------------------------------------------------------------------------------- -------------------------------------------- VOCABULARY Q.1-4.Below is given a single word with options to its meaning. You have to select all those options which are synonyms/antonyms of the word. Select the correct alternative from (1), (2), (3), (4) and (5) which represents all those synonyms/antonyms.
  • 18. Q.1. Turbulence A. commotion B. pandemonium C. fuss D. confusion (1) Only C (2) Only D (3) Only C and D (4) All except B (5) All of the above Q.2. Vigilant A. wary B. pernicious C. circumspect D. captive (1) Only A (2) Only A and C (3) Only B and D (4) Only B and C (5) All of the above Q.3. Deficit A. hierarchy B. Inadequacy C. scantiness D. paucity (1) Only D (2) Only B and C (3) Only C and D (4) All except A (5) All of the above Q.4. Fallacious A. mute B. illogical C. pursue D. shrewd (1) Only A (2) Only B (3) Only C (4) Only A and B (5) All of the above Answer Q.1.(5) Q.2.(2) For other options: Pernicious (adj)-hurtful Captive (n)- prisoner Q.3.(4) For other options: Hierarchy (n)- order Q.4.(2) For other options: Mute (adj)- silent Shrewd (adj)-clever --------------------------------------------------------------------------------------------------------------------------------------- -------------------------------------------- CLOZE TEST Q.1-10.In the following passage there are blanks each of which has been numbered. These numbers are printed below the passage and against each five words have been suggested, one of which fits the blanks appropriately. Find out the appropriate word in each case. The empty ‘Chyawanprash’ containers near the makeshift kitchen at the elephant camp say it all – that the elephants have been having a healthy, nutritious diet as (1) by the veterinary doctors. The camp managers say that the elephants get to have the nutritious (2) twice a day – morning before bath and early evening, walk eight to 10 km on the walking track twice a day and have loads of green fodder. A few of these elephants also undergo special medication, if necessary. ‘Valli’, a female elephant from the Koodal Azhagar Perumal Koil, Madurai, and ‘Vedanayagi’ another female elephant from Bhavani, Erode, are two such elephants. Based on the veterinarians’ prescription, the managers are treating the two for foot rot disease.
  • 19. The elephants get to (3) their legs in a decoction of seven chemicals, a traditional formula, to get over the problem. The foot rot sets in when the elephants are obese, or stand on hard surface or bitumen- topped roads for long with very little movement. Likewise, two other elephants are also undergoing eye treatment to overcome the ‘watery eye’ problem. This occurs when the elephants’ living (4) is hot. The managers say that the ingredients of the food and the quantity given to the elephants (5) from one to another and are dependent on the age and gender. Based on the two, a body-mass-index of sorts is derived and that determines the food and the quantity. Right at the start of the camp, the managers have noted down the weight of each elephant. This will be compared to the (6)that they will record when the elephants exit the camp around the second week of January. The managers say that one important factor in the camp is giving green fodder, which the elephants (7) in plenty in the camp. For the weak elephants, the camp managers give twigs of ‘aal’, ‘arasu’ ‘athi’ trees and also ‘koondapanai’. They add that the managers are also (8) the mahouts and those accompanying the elephants on the ways to keep the animal healthy. This is(9) the mahouts are with the elephants 24x7. And also because the animal should continue to live in a (10) environment. Q.1. (1) assured (2) prescribed (3) decided (4) convinced (5) established Q.2. (1) calories (2) starvation (3) greenery (4) diet (5) fast Q.3. (1) immerse (2) sprinkle (3) saturate (4) drown (5) bury Q.4. (1) envelope (2) appearance (3) effect (4) presence (5) atmosphere Q.5. (1) dissent (2) turn (3) vary (4) displace (5) remain Q.6. (1) gravity (2) load (3) substance (4) weight (5) value Q.7. (1) get (2) capture (3) earn (4) realize (5) access Q.8. (1) brainwashing (2) developing (3) Educating (4) asking (5) enriching Q.9. (1) never (2) whatever (3) however (4) since (5) because Q.10. (1) sound (2) strong (3) fresh (4) healthy (5)safe ANSWERS Q.1.(2) Prescribed For other options: Convinced (v)- explained Established (v)- proved Q.2.(4) Diet For other options: Starvation (n)- being hungry Greenery (n)- green with vegetation Fast (n)- staying hungry willingly Q.3.(1) Immerse For other options: Sprinkle (v)- spread Bury Q. 4.(5) Atmosphere For other options:
  • 20. Envelope (n)-cover Presence (n)-existence Q.5.(3) Vary For other options: Dissent (n)- disagreement Displace (v)- remove Remain (v)- be left Q.6.(4) Weight For other options: Gravity (n)-force Load (n)- burden Q.7.(1) Get For other options: Capture (v)- hold Access (v)- reach Q.8.(3) Educating For other options: Brainwashing (v)- changing thoughts Enriching (v)- making better Q.9.(5) Because Q10.(4) Healthy For other options: Sound (adj)-safe --------------------------------------------------------------------------------------------------------------------------------------- ---------------------------------------------- VOCABULARY Q.1- 5.In each of the following question four words are given of which two words, are most nearly the same or opposite in meaning to the given word. Find the two words which are most nearly the same or opposite in meaning and indicate the number of the correct letter combination, by darkening the appropriate oval in your answer sheet. Q.1. Scanty A. exiguous B. discrete C. meager D. scarce (1) Only C (2) Only D (3) Only A and D (4) All except B (5) All of the above Q.2. Accused A. dilemma B. implicated C. charged D. transient (1) Only A and C (2) Only B and C (3) Only B andD (4) All except A (5) All of the above Q.3. Panic A. trepidation B. scare C. frenzy D. consternation (1) Only A (2) Only B (3) Only C (4) Only D (5) All of the above
  • 21. Q.4. Elusive A. tricky B. vigilance C. vandalize D. deceptive (1) Only B (2) Only D (3) Only A and D (4) Only B and C (5) All of the above Q.5. Limit A. fracas B. bizarre C. restrain D. wanton (1) Only A (2) Only C (3) All except A (4) All except D (5) All of the above ANSWERS Q.1.(4) For other options: Discrete (adj)- different Q.2.(2) For other options: Dilemma (n)- a difficult situation Transient (adj)-temporary Q.3.(5) Q.4.(3) For other options: Vandalize (v)- cause destruction Q.5.(2) For other options: Fracas (n)- disturbance Bizarre (adj)- strange Wanton (adj)- careless --------------------------------------------------------------------------------------------------------------------------------------- ---------------------------------------------- COMPREHENSION Q.1-7.Read the following passage carefully and answer the questions given below. Caste restricts opportunity. Restricted opportunity constricts ability. Constricted ability further restricts opportunity. Where caste prevails, opportunity and ability are restricted to ever-narrowing circles of the people. Today, India on the one hand is on the verge of launching its second space shuttle to the moon; on the other hand, its citizens are 'confusion personified'. Amidst all this chaos about caste, creed, vote banks, terrorism and much other worldly ridicule, it is time for us to retrospect and take pragmatic steps towards building a casteless society. Protesting against conducting a caste-based census would hardly solve the issue, for shunning ourselves from the truth won't make the bitter truth sweet. A country which has suffered the caste system for over a thousand years can't become 'casteless' overnight. A caste-based census was extremely important in a country like ours given the reservations and the quotas that we have to plan. So, instead of criticizing the system, we need to find a solution within the system which would lead us to our goal. The Indian Constitution doesn't give us an option not to belong to any caste. Our society does, but not the Constitution. An entirely personal and spiritual endeavour such as this definitely doesn't need a
  • 22. government seal, but the lives and thoughts of the great people who preached secularism failed to become a movement because of the above mentioned flaw in our Constitution. If an idea gets constitutionalised or gets governmental back-up, it surely will gain momentum. It would undoubtedly be a brave decision to enroll oneself as 'casteless', considering the opposition and criticism these people have to face from their families, and only an intellectually cultured mind can do that. So be it a 'dalit' or an upper caste person, who elevates himself as 'casteless', because he is educated, will definitely not need any kind of reservation and he falls into the general category (no quotas, mind you). There will be some practical problems. Now, because of this categorical shift, there will be need for some alterations in the reservation percentage and a few other minor practical issues, to which solutions can be worked out. But the advantages of this system seem promising. The children of these 'casteless' parents will, by birth, have nothing tagged to their names, and this continues, generation to generation. Isn't it wonderful to see a tree pop up amidst acres and acres of weed? And won't this tree produce more trees? One day, can't we see a thick forest in that place? A thousand years later, if even this idea gets corrupted, someone else will find a better solution. As of now, if being casteless can become a constitutional provision, our society will definitely raise itself a few notches higher. Q.1. Why has the example of ' a tree popping up amidst acres of weed' been cited by the author? (1) The best of individuals come from socially backward families (2) Trees which grow in weed do not survive for long (3) To convey that a small effort from an individual triggers a big movement (4) An individual alone cannot do much to change the society (5) Not clear in the passage Q.2. What is meant by 'confusion personified' as mentioned in the passage? (1) Confusion prevails in the matters related to government (2) Society confuses people with its self made concepts (3) People remain confused about their own caste (4) People remain confused about every small or big issue (5) All of the above Q.3. Which of the following is the author's concern? (1) The children of 'casteless parents' will have great problem earning their living (2) The Indian constitution should provide an option to criticize caste system (3) The case system is nullifying the effect of the progress India has made in different fields (4) In future being casteless would mean being 'socially backward' (5) Not mentioned in the passage Q.4. Why, according to the passage, a caste-based census inevitable in the Indian context? (1) India is a nation which offers shelter to people coming from all castes and origins (2) Due to the provision of reservations and quotas (3) Because of the superstition and prejudices people do not get rid of. (4) The politicians exploit emotions of the people to their best advantage (5) Not mentioned in the passage Q.5. Which of the following is not true about the person who enrolls himself as 'casteless'? A. He would be discarded by other members of the society B. Nothing would be tagged to the names of his children C. He would not demand reservation D. His financial independence would be gone as soon as he declares himself 'casteless
  • 23. (1) Only B (2) Only D (3) Only A and B (4) Only A and D (5) All of the above Q.6. What notion does Indian constitution hold about 'caste system'? (1) It is the foundation pillar of many Indian societies (2) However hard we may try but it cannot be uprooted (3) It is required to prove one's identity in societal aspect (4) It is a means for the socially backward people to move ahead in any field (5) Not mentioned in the passage Q.7. Which of the following is TRUE in the context of the passage? (1) Where caste prevails, opportunity and ability are allowed (2) Our society would get better if the idea of being casteless becomes a constitutional provision (3) The Indian Constitution provides us an option not to belong to any caste (4) It is difficult to come up with a better solution to the problem of caste system (5) All of the above ANSWERS Q.1.(3) Q.2.(4) Q.3.(3) Q.4.(2) Q.5.(4) Q.6.(5) Q.7.(2) --------------------------------------------------------------------------------------------------------------------------------------- ---------------------------------------------- ERROR DETECTION Q.1-5. Read each part of the sentence to find out if there is any error in it. The error, if any, will be in one part of the sentence. The number of that part is the answer. If there is no error, mark your answer as (5). Q.1. A new system to monitor (1)/ the attendance of government school teachers(2)/ is being introduced (3)/in the district (4)/ No error (5) Q.2. Labourers are busy applying white cement (1)/on the tombs of the palace (2)/after completions of which (3)/ they would paint it afresh (4)/ No error (5) Q.3. The employees are expected (1)/to plan their expenditure (2)/and avail loans (3)/prudently and responsibly(4)/ No error (5) Q.4. The poster content provoked (1)/one section of a community (2)/ and the police were deployed (3)/at the area to control the situation (4)/ No error (5) Q.5. The international stadium (1/) would offer good facilities (2/)and opportunities to both (3)/national and international level sportsperson.(4)/ No error (5) ANSWERS Q.1.(5) Q.2.(3) ‘Completion’ would be used – ‘completion’ is an abstract noun which can never have a plural form
  • 24. Q.3.(3) ‘Avail’ should be followed by ‘of’ Q.4.(4) ‘In’ should replace ‘at’- to make the sentence correct Q.5.(5) --------------------------------------------------------------------------------------------------------------------------------------- ---------------------------------------------- SENTENCE IMPROVEMENT Q.1-5.Which of the phrases (1), (2), (3) and (4) given below should replace the phrase given in bold in the following sentence to make the sentence grammatically meaningful and correct. If the sentence is correct as it is and ‘No correction is required’, mark (5) as the answer. Q.1. He behaved though it was his fault, but we knew he was not responsible for it. (1) even though it was (2) though it was not (3) as if it was (4) despite it was not (5) No correction required Q.2. She never felt that it was not of her business to get involved in somebody else’s family matter. (1) were not of her business (2) was none of her business (3) was of not her business (4) was not of her businesses (5) No correction required Q.3. Being born in a certain family is not in our control. (1) Be born (2) Taking born (3) By birth (4) Being borned (5) No correction required Q.4. I was taken back by his sudden comment on this issue. (1) would be taken back by (2) was taken backwards by (3) was taken back for (4) was taken aback by (5) No correction required Q.5. In a matter of seconds, we come to know of what is happening anywhere in the world. (1) came to know of (2) come to be known of (3) come to know off (4) are coming to know of (5) No correction required ANSWERS Q.1.(3) Use ‘as if it was’ in place of ‘though it was’.
  • 25. Q.2.(2) Use ‘was none of her business’ in place of ‘was not of her business’. Q.3.(5) Q.4.(4) Use ‘was taken aback by’ in place of ‘was taken back by’. Q.5.(5) --------------------------------------------------------------------------------------------------------------------------------------- ---------------------------------------------- FILL IN THE BLANKS Q.1-5. In each of the following sentences there are two blank spaces. Below each five pairs of words have been denoted by numbers (1), (2), (3), (4) and (5). Find out which pair of words can be filled up in the blanks in the sentences in the same sequence to make the sentence meaningfully complete. Q.1. The athletes have been able to establish how the ——————substance——————— their body. (1) prohibit, enters (2) prohibits, enter (3) prohibiting, entering (4) prohibit, enter (5) prohibited , entered Q.2. The judges made it clear that they were not —————to pass —————direction. (1) apprehension, any (2) divided , many (3) inclined , any (4) ruled , some (5) curiosity , much Q.3. HRD ministry issued a detailed gazette notification ——————all the central educational institutions to abide ———the decision. (1) asking, by (2) demanded, from (3) revert, to (4) applaud, at (5) giving , in Q.4. They have been requesting the authority for —————sometime, but no ——————response has come through. (1) quiet, desire (2) quite , positive (3) quiet , authority (4) quit, negativity (5) quite , potential Q.5. ———————yarn available at a low price had been one of the —————demands of the weavers. (1) Doing, measure (2) Implementing, best (3) Happening , popularity (4) Making , major (5) Associating , most ANSWERS Q.1.(5) prohibited , entered Q.2.(3) Inclined , any For other option: Apprehension- fear Q.3.(1) asking , by For other options: revert- to reply applaud- to express approval Q.4.(2) quite , positive For other options:
  • 26. Authority – person having power Potential - having strength Q.5.(4) Making , major For other options: implementing – to put to practice associating- connecting --------------------------------------------------------------------------------------------------------------------------------------- ---------------------------------------------- REARRANGEMENT Q.1-5. Rearrange the following six sentences (A), (B), (C),(D) and (E) in the proper sequence to form a meaningful paragraph; then answer the questions given below them. (A) Without a productive labour force. (B) Among the natural resources which can be called upon in natural plans for development. (C) Including effective leadership and intelligent middle management. (D) Possibly the most important is human labour. (E) No amount of foreign assistance or of natural wealth can ensure successful development. Q.1. Which of the following should be the SECOND sentence after rearrangement? (1) A (2) B (3) C (4) D (5) E Q.2. Which of the following should be the FOURTH sentence after rearrangement? (1) A (2) B (3) C (4) D (5) E Q.3. Which of the following should be the FIRST sentence after rearrangement? (1) A (2) B (3) C (4) D (5) E Q.4. Which of the following should be the Last sentence after rearrangement? (1) A (2) B (3) C (4) D (5) E Q.5. Which of the following should be the THIRD sentence after rearrangement? (1) A (2) B (3) C (4) D (5) E ANSWERS B is the first sentence after rearrangement because only B is independent .B and D is mandatory pair A continues the idea given in B and D so it is 3rd sentence and A,C is mandatory pair, and E ends the topic so it is last. Q.1.(2) B Q.2.(4) D Q.3.(1) A Q.4.(3) C Q.5.(5) E The correct sequence is BDACE
  • 27. --------------------------------------------------------------------------------------------------------------------------------------- ---------------------------------------------- MISAPPROPRIATION Q.1-5. In each question below a sentence with four words printed in bold type is given. These are numbered (1),(2),(3) and (4) . One of these four words printed in bold may be either wrongly spelt or inappropriate in context of the sentence. Find out the word which is wrongly spelt or inappropriate if any. The number of that word is your answer. If all the words printed in bold are correctly spelt & also appropriate in the context of the sentence, mark (5) “All correct” as your answer. Q.1. The Economic upheaval in one part of the world has direct bearing on the economic activties (1) (2) (3) (4) across the globe. All correct. (5) Q.2. All parts of the world are tied to each other inextricable through electronic media. All correct (1) (2) (3) (4) (5) Q.3. At the summit, extensive discussions were held on the challenges currently being faced by the (1) (2) (3) human society which include debt and sustainabilyty. All correct (4) (5) Q.4. The world leaders also shed light on various other partinant issues facing the world at present. (1) (2) (3) (4) All correct (5) Q.5. Renuable energy technologies have made significant contributions to solving some of the most (1) (2) (3) urgent problems the world is facing. All correct (4) (5) ANSWERS Q.1.(4) Activities in the correct spelling Q.2.(3) Inextricably should be used Q.3.(4) Sustainability is the correct spelling Q.4.(3) Pertinent is the correct spelling Q.5.(1) Reviewable is the correct spelling --------------------------------------------------------------------------------------------------------------------------------------- ---------------------------------------------- SENTENCE IMPROVEMENT Q.1-5. In each question below a sentence with four words printed in bold type is given. These are numbered (1),(2),(3) and (4) . One of these four words printed in bold may be either wrongly spelt or inappropriate in context of the sentence. Find out the word which is wrongly spelt or inappropriate if any. The number of that word is your answer. If all the words printed in bold are correctly spelt & also appropriate in the context of the sentence, mark (5) “No error” as your answer.
  • 28. Q.1. The red blossoms accented the prevailing pink of the house front. No error (1) (2) (3) (4) (5) Q.2. Our strongest reason for siding with the communist is because our father is a communist. (1) (2) (3) (4) No error (5) Q.3. He ordered the servants to prepare a feast had the gates opened and to fire a salute. (1) (2) (3) (4) No error (5) Q.4. The pilot revealed that his airbus was carrying not less than three hundred passengers (1) (2) (3) last evening. No error (4) (5) Q.5. The nineteenth Ammendement to the constitution gave women the right to vote in the (1) (2) (3) elections of 1920. No error (4) (5) ANSWERS Q.1.(2) Use ascented in place of accented. Q.2.(4) Use ‘that’ in place of because as the word ‘reason’ has already been used. Q.3.(3) Use “when” in place of had Q.4.(3) Write more than in place of less than Q.5.(1) The correct spelling is amendment. --------------------------------------------------------------------------------------------------------------------------------------- ---------------------------------------------- ERROR DETECTION Q.1-5. Read each sentence to find out if there is any error in it. The error, if any, will be in one part of the sentence. The number of that part is the answer. If there is no error, the answer is (5). Q.1. While one part of the TV programme carried the football game ,the (1)/ other part shows the (2)/ training(3)/ of the tennis(4)/ No Error(5) Q.2. Prior to the examination(1) / the instructor had told us to keep (2)/ an eye on the time to(3)/ write clearly and no cheating(4) /No Error (5) Q.3. The rainy season(1)/ had already began(2)/ by the time they arrived(3)/ in Burma that year(4)/ No Error (5) Q.4. Whenever these three (1)/ eminent scientist spoke to(2)/ each other an argument(3)/ was sure(4) No Error (5) Q.5. There was scarcely(1)/ on time given to think about(2)/the problem before the bell (3)/at the end of class(4)/ No Error(5) ANSWERS
  • 29. Q.1.(3) "Showed" should replace "shows because the sentence is in past tense." Q.2.(2) "Not to cheat" should replace "nocheating because and connects the safe same from of verb." Q.3.(2) "Begun"should replace "begun"because had is followed by v3 from. Q.4.(3) "One another" should replace"each other" because one another is used for more than two persons. Q.5.(2) "Any" should replace "no" because scascely already given negative sense and double negative is not used in a sentence. --------------------------------------------------------------------------------------------------------------------------------------- ---------------------------------------------- FILL IN THE BLANKS Q.1-5. In each of the following sentences there are two blank spaces. Below each five pairs of words have been denoted by numbers (1), (2), (3), (4) and (5). Find out which pair of words can be filled in the blanks inorder to make a meaningful sentence. Q.1. Football evokes a ________ response in India compared to cricket, that almost _____the nation. (1) magnifying, manipulates (2) fascinating, forbids (3) lukewarm, electrifies (4) wild, shakes (5) tentative, guides Q.2. The cigarette manufactures’ association has _________ to _________in aggressive promotion. (1) decided, indulge (2) calculated, call (3) forgotten, motivate (4) predicted, plant (5) cared, sell Q.3. Social studies, Science, ______of health and safety, the very atmosphere of classroom, are few of the important areas for the ______of proper emotional reactions. (1) course, clearance (2) matters, formation (3) areas, rulings (4) radius, spirit (5) mature, provision Q.4. The chief idea of one very ________type of traveler is to see as many objects of ________ as he possibly can. (1) measure, hate (2) minor, power (3) error, fun (4) yielding, utility (5) common, interest Q.5. Learning is more efficient when it is _______. It is less efficient when it is _____. (1) fast, slowly (2) rapidly, turtle-slow (3) tedious, obedient (4) fun, drudgery (5) growing, imparting ANSWERS Q.1.(3) Lukewarm- indifferent Electrifies- thrill, amaze For other options: Magnifying- enlarging, intensifying Manipulates- maneuver Fascinating- interesting, spellbinding Forbid- prohibit an action Tentative- conditional, experimental
  • 30. Q.2.(1) decided, indulge Indulge- perform service Predicted- express an outcome in advance Q.3.(2) matters, formation Q.4.(5) common. Interest For other options: Yielding- accommodating Q.5.(4) fun, Drudgery Drudgery- hard, tedious work For other options: Turtle-slow- very slow --------------------------------------------------------------------------------------------------------------------------------------- ---------------------------------------------- COMPREHENSION Q.1-10.Read the following passage carefully and answer the questions given below it. Certain words have been printed inbold to help you locate them while answering some questions. The Reserve Bank of India (RBI) stipulated that debit cards would be issued to customers having Savings Bank and Current Accounts but not to cash credit or loan account holders. Banks may issue only online debit cards, including co-branded debit cards where there is an immediate debit to the customers’ account, and where straight through processing is involved. Banks are, henceforth, not permitted to issue offline-debit cards. Banks which are now issuing offline debit cards may conduct a review of their offline debit card operations and discontinue operations of such cards within a period of six months from the date of the circular issued. Banks were also asked to ensure that customers were duly informed regarding switching over to online debit cards. However, till such time as offline cards were phased out, the outstanding balances / unspent balances stored on the cards would be subject to computation of reserve requirements. As per the notification, no bank shall dispatch a card to a customer unsolicited, except in the case where the card is a replacement for a card already held by the customer. Each bank shall make available to the cardholders in writing, a set of contractual terms and conditions governing the issue and use of such a card. These terms shall maintain a fair balance between the interests of the parties concerned and shall be expressed clearly. The terms shall specify that the bank shall be responsible for direct losses incurred by a cardholder due to a system malfunction directly within the bank’s control. However, banks would not be held liable for any loss caused by a technical breakdown of the payment system if the breakdown of the system was recognizable for the cardholder by a message on the display of the device or otherwise known. The responsibility of the bank for the non-execution or defective execution of the transaction is limited to the principal sum and the loss of interest subject to the provisions of the law governing the terms. With a view to reducing the instances of misuse of lost/stolen cards, the apex bank asked banks to issue cards with photographs of the cardholder or any other advanced methods that may evolve from time to time. Banks should undertake review of their operations/issue of debit cards on half-yearly basis. The review would include, inter-alia, card usage analysis, including cards not used for long durations due to their inherent risks. Banks were asked to ensure to put in place an effective mechanism for redressal of customer complaints. The name, designation, address and contact number of important executives as well as the grievance redressal officer of the bank may be displayed on the website. Also, there should be a system of acknowledging customers’ complaints for follow-up, such as complaint number / docket number. Q.1. Why have banks been asked to discontinue their offline debit card operations? (1) Offline debit card issuing system has been facing problems for quite sometime
  • 31. (2) As per the new circular, debit cards will be issued online only (3) Banks should make use of the latest technology available (4) With offline debit card system any transaction takes a long time. (5) None of these Q.2. Which of the following has been referred as ‘Apex bank’ in the passage? (1) Bank Issuing the online debit card (2) Bank which still offers offline debit cards (3) Every bank which is governed by RBI (4) Reserve Bank Of India (5) None of these Q.3. Which of the following is/are essential to be displayed on website for redressal of customer complaints? A. Complaint number / docket number B. Complaint type C. Details of grievance redressal officer of the bank D. Details of important executives (1) Only C (2) Only A and C (3) All except B (4) All except A (5) All of the above Q.4. How much time have the banks been given to discontinue operations of offline debit cards ? (1) 1 year (2) 3 months (3) 1 month (4) 6 months (5) 2 weeks Q.5. Which of the following is FALSE, according to the passage? (1) Every year Banks should undertake review of their operations/issue of debit cards. (2) Banks should resort to advanced methods to reduce the instances of misuse of lost/stolen cards (3) Bank would be responsible for direct losses incurred by a cardholder because of a system malfunction directly within the bank’s control (4) Each bank should make available to the cardholders in writing, a set of contractual terms and conditions governing the issue and use of a debit card (5) All of the above Q.6. Which of the following should maintain a fair balance between the interests of the parties concerned, as per the notification? (1) Contractual terms (2) Circular issued (3) Review of banks’ operations/issue of debit cards (4) System of acknowledging customers’ complaints (5) Not mentioned in the passage Q.7-8.Choose the word most similar in meaning to the word printed in bold, as used in the passage. Q.7. Liable (1) Responsible (2) Innocent (3) Caught (4) Inclined (5) Disciplined Q.8. Duly (1) Suitably (2) Equally (3) Nicely (4) Properly (5) Liberally Q.9-10.Choose the word which is most nearly the OPPOSITE in meaning as the word printed in bold as used in the passage. Q.9. Inherent
  • 32. (1) Integral (2) External (3) Essential (4) Incidental (5) Original Q.10. Dispatch (1) Hurry (2) Send (3) Receive (4) Perform (5) Destroy ANSWER Q.1.(2) Q.2.(4) Q.3.(3) Q.4.(4) Q.5.(1) Q.6.(1) Q.7.(1) Responsible For other options: Inclined (v)- interested Q.8.(4) Properly For other options: Liberally (adv)- with liberty Q.9.(2) External For other options: Essential (adj)- important Incidental (adj)- by chance Q.10.(3) Receive --------------------------------------------------------------------------------------------------------------------------------------- ---------------------------------------------- REARRANGEMENT Q.1-5. Rearrange the following seven sentences (A), (B), (C), (D), (E), (F) and (G) in the proper sequence to form a meaningful paragraph; then answer the questions given below. (A) But seriously, how much would you pay to know what thoughts are swimming around in someone else’s head? (B) In most fictional movies, thus, the idea of reading minds-of seeing the private intentions of another, and the possibility of intervening in those plans - has always been highly attractive. (C) Such fantastic questions have long been the bread and butter of fiction, (D) Today, more than four centuries since the phrase, “A penny for your thoughts?”, was first recorded, inflationary accounting makes that ancient penny was worth more than $40. (E) The going rate for a “thought”-a probe into the thinking of another-was once quite a bargain. (F) And if you could really know their truthfulness, how much more would you pay? (G) Even with the sliding value of the dollar, this still seems quite a bargain. Q.1. Which of the following should be the SECOND sentence after rearrangement? (1) F (2) D (3) C (4) G (5) B Q.2. Which of the following should be the FOURTH sentence after rearrangement?
  • 33. (1) A (2) B (3) G (4) D (5) F Q.3. Which of the following should be the SIXTH sentence after rearrangement? (1) F (2) G (3) C (4) B (5) A Q.4. Which of the following should be the SEVENTH (Last) sentence after rearrangement? (1) A (2) B (3) C (4) D (5) E Q.5. Which of the following should be the FIRST sentence after rearrangement? (1) A (2) B (3) C (4) D (5) E ANSWERS Q.1-5. The correct rearrangement is EDGAFCB Q.1.(2) D Q.2.(1) A Q.3.(3) C Q.4.(2) B Q.5.(5) E --------------------------------------------------------------------------------------------------------------------------------------- ---------------------------------------------- ERROR DETECTION Q.1-5. Read each part of the sentence to find out if there is any error in it. The error, if any, will be in one part of the sentence. The number of that part is the answer. If there is no error, mark your answer as (5). Q.1. Medicines for treatment of tuberculosis (1)/will now be available (2)/for free at all (3)/chemical shops and corporate hospitals. (4)/ No error (5) Q.2. A young community activist began a road journey (1)/though 21 countries to emphasize(2)/ the need for peace (3)/ between India and Pakistan (4)/ No error (5) Q.3. Urdu language played (1)/a pivotal role (2)/ in national integration (3)/during the war of independence. (4)/ No error (5) Q.4. Without waiting for(1)/ the instructions from the government(2)/ some cable operators in (3)/ the city had block the news. (4)/ No error (5) Q.5. A long coastline means(1)/ better opportunities for (2)/ fishing which solves unemployed (3)/ and food problems to a certain extent (4)/ No error (5) ANSWERS Q.1.(5) Q.2.(2) ‘Through’ should replace ‘to’ to make the sentence meaningful Q.3.(5)
  • 34. Q.4.(4) use ‘blocked’ in place of ‘block’. Q.5.(3) use ‘unemployment’ instead of ‘unemployed’. --------------------------------------------------------------------------------------------------------------------------------------- ---------------------------------------------- HOMONYMS Q.1-5.In each question, there are three sentences. Each sentence has pairs of words/phrases that are italicized and highlighted. From the italicized and highlighted word(s)/ phrase(s), select the most appropriate word(s)/ phrase(s) to form correct sentence. Then, from the options given, choose the best one. Q.1. Characters tie events (A)/ invents (B) in a story together and provide a thread of continuity and meaning. His close confidant (A)/ confident (B) knew all of his secrets and was not ready to help us. Though (A)/ Through (B) known for its artistic merits, the film nonetheless did poorly at the box office. (1) AAA (2) BBB (3) ABA (4) BAB (5) AAB Q.2. The meeting ended without any plans for farther (A)/ further (B) discussions. As a result, everyone treats the other as an adversely (A)/ adversary (B) rather than an ally. He studied the creation, inhabitation (A)/ inhibition (B) and demise of the colony. (1) AAA (2) BBB (3) BBA (4) AAB (5) BAB Q.3. She prepared an apple pie for the desert (A)/ dessert (B) today. Avocado pear (A)/ peer (B) trees usually require sub tropical conditions to flourish. Amount of commission earned will vary (A)/ very (B) between employees. (1) AAB (2) BAA (3) ABA (4) BAB (5) AAA Q.4. Beside (A)/ Besides (B) being difficult, grammar can be confusing. There are hundreds of children around the country who will be adversely affected (A)/ effected (B) by this ruling. The balance of power will shift to the East as China and India involve (A)/ evolve (B). (1) BAB (2) ABA (3) AAB (4) BBA (5) BAA Q.5. It was a tough situation and Manasi was taking pain (A)/ pane (B) to make it better. The police, during the raid, ceased (A)/ seized (B) all the arms and ammunitions in their possession. The principal (A)/ principle (B) scolded the students for creating chaos during the recess. (1) AAA (2) BBB (3) BAB (4) AAB (5) ABA ANSWERS Q.1.(1) AAA Q.2.(3) BBA Q.3.(2) BAA Q.4.(1) BAB Q.5.(5) ABA
  • 35. --------------------------------------------------------------------------------------------------------------------------------------- -------------------------------------------- CLOZE TEST Q.1-10.In the following passage there are blanks each of which has been numbered. These numbers are printed below the passage and against each five words have been suggested, one of which fits the blanks appropriately. Find out the appropriate word in each case. Social networking sites are immensely useful in (1) boundaries when spreading social messages. But the flip (2)of social networking sites is their (3) for spreading destruction and social malaise. The latter trend was seen in two recent uses of social networking. The first one was of a man selling his grandchild to a businessman in Delhi for Rs 8 lakh after striking a deal using Facebook. The other was the Dow Jones falling by 143 points after hackers sent a message — ‘Two Explosions in the White House and Barack Obama is injured’ — from the Twitter handle of a news organization. The Syrian Electronic Army, which had (4) the Reuters feed last year, took credit for this. Social networking sites have shrunk the world and connected the most disparate of people like never before. However, it is the challenges (5) by its dark underbelly that governments around the world have not been able to control. China, despite it denying any part in it, has been (6) in its cyber attacks on countries and organizations that have spoken against its interests. A recent example of Beijing’s use of cyber space to (7) unpalatable views was the attack on the New York Times’ website after it ran a feature exposing former premier Wen Jiabao’s wealth. Many countries, including India, have been victims of Chinese hackers. Al-Qaeda has been using the Internet, especially video-sharing platforms and social networking sites, for (8) and recruitment. To think that a single tweet can affect international markets and in other cases escalate tension between two nations is spine chilling. Does this mean that the government should roll down the shutters on social networking sites? Definitely not. Real-time censoring of Internet traffic, given the volume (which is only going to increase), is unthinkable. Governments, especially in India, should, through its various wings, create more Internet awareness and give, cyber monitoring cells in the police and other agencies ,more teeth. Internet and social networking sites are here to stay and evolve; shying away from them or banning them will be (9). Governments should be one step ahead of forces that exploit these platforms for (10) purposes. This is the only way it can really get a handle on things. Q.1. (1) migrating (2) replicating (3) transcending (4)accomplishing (5) organizing Q.2. (1) turn (2) schedule (3) cases (4) corner (5) side Q.3. (1) potential (2) inherent (3) sluggish (4) latency (5) litigation Q.4. (1) concerned (2) hacked (3) captivated (4) overwrought (5) irate Q.5. (1) masquerade (2) preferred (3) juxtaposed (4) posed (5) transformed Q.6. (1) notorious (2) obvious (3) adorable (4) chosen (5) salient Q.7. (1) utilize (2) welcome (3) assist (4) hoodwink (5) stymie Q.8. (1) publication (2) brainstorming (3) propaganda (4) fundamental (5) tradition Q.9. (1) intellectua (2) counterproductive (3) miserably (4) idle (5) vainglorious
  • 36. Q.10. (1) ethical (2) voluminous (3) lawful (4) virtuous (5) nefarious ANSWERS Q.1.(3) Transcending For other options: Migrating (v)-traveling to another place Replicating (v)- copying Accomplishing (v)- completing Q.2.(5) Side Q.3.(1) Potential For other options: Inherent (adj)-elementary Sluggish (adj)-inactive Latency (n)-suspension Litigation (n)-matter coming before court Q.4.(2) Hacked For other options: Overwrought (adj)- exhausted and excited Irate (adj)- angry Q.5.(4) Posed For other options: Masquerade (n)-disguise Juxtaposed (v)- place side by side Q.6.(1) Notorious For other options: Obvious (adj)- clear Adorable (adj)-lovable Salient (adj)- prominent Q.7.(5) Stymie For other options: Hoodwink (v)- deceive Q.8.(3) Propaganda For other options: Fundamental (adj)- basic Q.9.(2) Counterproductive For other options: Miserably (adv)- badly Vainglorious (adj)-boastful Q.10.(5) Nefarious For other options: Voluminous (adj)-vast Virtuous (adj)-ethical
  • 37. --------------------------------------------------------------------------------------------------------------------------------------- -------------------------------------------- FILL IN THE BLANKS Q.1-5.In each of the following sentences there are two blank spaces. Below each five pairs of words have been denoted by numbers (1), (2), (3), (4) and (5). Find out which pair of words can be filled in the blanks inorder to make a meaningful sentence. Q.1. A day —— call is one which tells you to buy or ————— a share on the same day. (1) trading, share (2) trading, sell (3) trades, sell (4) trading, buy (5) trading, close Q.2. It is ———— to read the SMS: “Make money trading equity derivatives up to 95% —————. (1) heartening, accurate (2) hard, only (3) heartening, approximation (4) hard, accurate (5) heartening, accuracy Q.3. The Food Corporation of India is —— to sell wheat ———— spot exchanges for bulk and retail purchasers. (1) making, through (2) planning, while (3) planning, through (4) making, across (5) making, within Q.4. The post - reforms ———— economic sentiment has —————— up pre-placement offers and pre-placement interviews at the country’s top non-IIM management institutes. (1) negative, fired (2) positive, fired (3) positive, shot (4) positive, given (5) positive, done Q.5. When the mid-term review of the employee’s performance does not ————— expectations, an escalation to senior managers may not help because often, senior managers are ——— in reviews. (1) meet, involve (2) work, indulged (3) meet, indulge (4) beyond, involved (5) meet, involved ANSWERS Q.1.(2) trading, sell Q.2.(5) heartening, accuracy For other options: Approximation (n)-estimation Q.3.(3) planning, through For other options: Across (prep)-in all parts of Through (prep)- by the medium of Q.4.(2) positive, fired For other options: Shot (v)- went up/ incresed Q.5.(5) Meet, involved For other options: Beyond (prep)- Over
  • 38. Indulge (v)- get into something --------------------------------------------------------------------------------------------------------------------------------------- --------------------------------------- REARRANGEMENT OF SENTENCES Q.1-5.Rearrange the following sentences (A),(B),(C), (D),(E),(F),(G) and (H) in the proper sequence to form a meaningful paragraph, then answer the questions given below them. (A) As one who had the good fortune of being in the company (B) The world of Indian classical music has become poorer (C) He was very kind at heart and would pacify them immediately (D) Of some of his disciples in the 1960s, I was wonderstruck by the preparation (E) He expected from his students before a concert. (F) We have lost a musician who could hold the audience spellbound (G) With the passing of the great sitar maestro, Pandit Ravi Shankar. (H) While he used to get annoyed with them at the slightest sign of imperfection Q.1. Which of the following is the SIXTH statement after rearrangement? (1) F (2) D (3) H (4) A (5) E Q.2. Which of the following is the FIRST statement after rearrangement? (1) E (2) D (3) H (4) B (5) A Q.3. Which of the following is the EIGHTH statement after rearrangement? (1) F (2) C (3) E (4) A (5) G Q.4. Which of the following is the SECOND statement after rearrangement? (1) G (2) D (3) C (4) E (5) B Q.5. Which of the following is the THIRD statement after rearrangement? (1) C (2) D (3) F (4) A (5) E ANSWERS Q.1.(5) Q.2.(4) Q.3.(2) Q.4.(1) Q.5.(3) The arrangement is BGFADEHC. The paragraph is about Pandit Ravi Shankar and the irreparable loss caused to the music world due to his death. Further, it has been mentioned he was a perfectionist and had the potential to charm the audience. --------------------------------------------------------------------------------------------------------------------------------------- ---------------------------------- MISAPPROPRIATION
  • 39. Q.1-8.In each set below five words have been printed which are numbered (1), (2), (3),(4) and (5). One of these words may be wrongly spelt.Find out the wrongly spelt word. The number of that word is the answer. If all the words are correctly spelt and are appropriate, the answer is (5) i.e. "All correct". Q.1. Inadequate employ skills and lack of confidence are some of the major hindrances these (1) (2) (3) children face in finding gainful employment All correct (4) (5) Q.2. The professor aprised the students about the scope of psychology and its role and functions in (1) (2) (3) our daily routine All correct (4) (5) Q.3. The real estate businessmen have created an artificial scarcity of land thereby making it (1) (2) (3) costly All correct (4) (5) Q.4. One of the biggest concerns for pet owners is the constraint that they cannot take their pets (1) (2) (3) out when they roam round in public places All correct (4) (5) Q.5. With stock markets on the upswing, iquity mutual funds have started to reward investors (1) (2) (3) (4) All correct (5) Q.6. All the elected representatives in the state would soon be able to availed the benefits of (1) (2) cashless treatment at selected government and private hospitals All correct (3) (4) (5) Q.7. An increasing number of accidents happen as cranes are driven along roads from (1) (2) one worksight to another even during peak hours All correct (3) (4) (5) Q.8. Weeding out fake ration cards is going on and once the work is completed,issuance of new (1) (2) (3) (4) cards would begin. All correct (5) ANSWERS
  • 40. Q.1.(1) ‘Employable ‘ should be used- an adjective is required at this place Q.2.(1) ‘Apprised’ is the correct spelling Q.3.(5) Q.4.(4) ‘Roaming around’ is used – which means ‘walking around for enjoyment’ Q.5.(3) ‘Equity’ is the correct spelling Q.6.(2) ‘Avail’ should be used – first form of verb should come after ‘to’ Q.7.(3) ‘Worksite’ is the correct word which means ‘site where the work is in progress’ Q.8.(5) --------------------------------------------------------------------------------------------------------------------------------------- --------------------------------------- COMPREHENSION Q.1-10.Read the following passage carefully and answer the questions given below it. Certain words have been printed inbold to help you locate them while answering some questions. During the last two decades, physicians and mental health professionals have begun to discover the limitationsof western allopathic medicine. The focus is on pathology and disease rather than on prevention. The destructiveness of so many pharmaceutical and surgical remedies, the separation of physical and emotional problems, theassumption of an asymmetrical relationship between an all- powerful physician and a submissive patient have led clinics and researchers to look for answers in other traditions and cultures. It is indeed disappointing in this context, that in India, with its long tradition of the body-mind continuum, health efforts are so largely dependent on the allopathic system which is now being increasingly rejected by western medical scientists. However, any wide open, fuzzy field like ‘holistic health’ offers abundant opportunities for fraud and overpromise. The new paradigm of health is not a licence for quackery. Holistic health is a perspective, not a specialisation or a discipline. The body-mind is a process, so is healing or making whole. It was discovered that it is not the simple physical change but rather the state of mind that is the key to health. This state has been called ‘restful alertness,’ ‘passive volition’, ‘deliberate betting’ etc. Cumulative stresses seem to melt under this seemingly paradoxical state, restoringnatural flow to the body-mind whirlpool. Clearly, health services, as we know them, are essential but, equally clearly, they are not enough. There is an urgent need to blend the ancient traditional ways with some of the discoveries of modern medicine to make way for a truly healthy society. The first and most important area deserving urgent attention is child health care. Conventionally, it is the doctor and the hospitals that are seen at the centre of all health care; the mother and the child are at the periphery. It is perhaps necessary to see the mother as the centre of child health care. It is she who is the highest level health worker not by training or in qualifications but in time and love, in the special knowledge of her own children, in the breadth of ‘integrated services’ she provides and in the permanent presence she brings to her child’s life. This indeed was the basis of the ancient child care practice and it is well that it is being rediscovered. The implications of this are manifold. Mother must be imparted the knowledge, skills and techniques required for her to do the job well. The old civilisations did this through word of mouth from one generation to another. Modern civilisations can improve on this but the essence will remain the same. Q.1. Which of the following is most likely in disagreement with the passage? (1) The western allopathic system of cure places less emphasis on prevention of disease and more on its treatment.
  • 41. (2) The western allopathic system is an extension of India’s ancient medical system. (3) The allopathic system is being rejected by Western medical scientists. (4) The allopathic system has takers in India. (5) All of the above Q.2. Which one of the following does not characterize the ‘holistic’ approach in medicine? (1) It is as susceptible to fraudulent practices as is any other approach. (2) Holistic health is a perspective and not a specialization or a discipline. (3) It is in tune with western culture and traditions. (4) None of the above (5) All of the above Q.3. Which of the following is in agreement with the passage? (1) A judicious blending of traditional and modern systems of medicine is what is required. (2) More research work needs to be done and facilities increased for western allopathic medicine to be successful. (3) The centre for child health-care is the primary health-care centre (4) The allopathic system of cure is increasingly being rejected by Indians (5) Both 3&4 Q.4. The centre of child health care should be (1) father (2) mother (3) doctor (4) hospitals (5) medical fraternity Q.5-7.Choose the word most similar in meaning to the word printed in bold, as used in the passage. Q.5. Limitations (1) Prohibition (2) Benefits (3) Constraints (4) Extent (5) Infinity Q.6. Assumption (1) Manipulation (2) Succession (3) Hindrance (4) Belief (5) Deduction Q.7. Imparted (1) Separated (2) United (3) Dragged (4) Pushed (5) Given Q.8-10. Choose the word which is most nearly the OPPOSITE in meaning as the word printed in bold as used in the passage. Q.8. Rejected (1) Declined (2) Supported (3) Insulted (4) Abandoned (5) Deserted Q.9. Quackery (1) Mischief (2) Honesty (3) Liberty (4) Sound (5) Dissatisfaction Q.10. Restoring (1) Saving (2) Accumulating (3) Adopting (4) Hampering (5) Improving ANSWERS
  • 42. Q.1.(2) The passage dwells on the weakness of the western allopathic system of cure. The emphasis is more on the pathology and disease than on prevention, so much so that the system is increasingly being rejected by western medical scientists. Q.2.(3) Q.3.(1) Q.4.(2) Q.5.(3) Constraints Limitations- disadvantages Q.6.(4) Q.7.(5) Given Q.8.(2) Supported Q.9.(2) Quackery- Deceitfulness, deception Q.10.(4) Hampering ---------------------------------------------------------------------------------------------------------------- ------------------------------------------------------------------- PARAGRAPH COMPLETION Q.1-5.In each of the following paragraphs there is one blank space. Below each sentence has been denoted by numbers (1), (2), (3), (4) and (5). Find out which sentence can be filled up in the blank to make the paragraph meaningfully complete. Q.1. It is difficult being a marketer in the digital age but now you feel you are on top of things. Your corporate website is finally optimised. ——————————————and have successfully grown communities on all of them. For those segments of your business that require mobile apps, you’ve developed apps straddling various operating systems. You’ve got the hang of digital out-of-home and have started building interactivity into your retail spaces as well. You have tied up two sets of agencies for your digital and physical strategies and are happy you roped in the absolute best in the business. (1) Customers today are bouncing in and out of multiple channels across different devices. (2) You have leveraged most of the social channels your customers are likely to use (3) Corporate budgets have yet to align to support all these channels (4) It is increasingly difficult to determine where in-store retailing ends and e-commerce begins. (5) Companies need to build an integrated communication strategy. Q.2. ——————————————now as the Indian Railways is working on a brand new ticketing website that promises to be three times faster than the existing IRCTC ticketing portal. The larger plan is to phase out the present IRCTC site. And if all goes well, the new portal would start functioning by November. In a step towards setting up a new portal, its Beta version (development stage) would be available on the www.irctc.co.in The users wanting a new booking experience different from the present IRCTC site can access it. (1) From the existing 2000 tickets per minute, the idea is to create a system (2) The aim is to achieve user delight through technology solution in railways (3) IRCTC books five lakh tickets per day (4) A separate journey planning would be available from the ticket booking stage so as to ease out the booking process. (5) Failed transactions during online booking of train tickets would be a thing of the past